ETHICAL LAWYERING FROM BOOK

¡Supera tus tareas y exámenes ahora con Quizwiz!

1. C. Under the ABA Model Rules, the applicable provision is comment 11 to Model Rule 1.7: "When lawyers representing different clients in the same matter or in substantially related matters are closely related by blood or marriage, there may be a significant risk that client confidences will be revealed and that the lawyer's family relationship will interfere with both loyalty and independent professional judgment. As a result, each client is entitled to know of the existence and implications of the relationship between the lawyers before the lawyer agrees to undertake the representation. Thus, a lawyer related to another lawyer, e.g., as parent, child, sibling or spouse, ordinarily may not represent a client in a matter where that lawyer is representing another party, unless each client gives informed consent." [See also ABA Formal Op. 340 (1975) (stating that the mere fact of marriage creates no actual conflict of interest, but the closeness of the wife-husband relationship does create opportunities for inadvertent violations of the ethics rules. For instance, one spouse might inadvertently learn confidential information when taking a telephone message for the other spouse at their home.)] In light of this, both lawyers must disclose the situation to their respective clients. If the two clients consent after this disclosure, then Cheryl and Dennis may participate on opposite sides of the case. [See Oregon State Bar Op. 502 (1984).]

1. After they graduated from law school, Cheryl and Dennis were married and went to work for separate law firms in a large city. Cheryl's practice is primarily trademark litigation, and Dennis's practice is primarily general business counseling; only rarely does he become involved in trial work. One of Dennis's regular business clients sued a major corporation for trademark infringement. Dennis and his law firm appear on the pleadings as counsel for plaintiff, but, in fact, all of the trial work is being done by another firm that specializes in trademarks. The defendant's lead counsel died suddenly, and his firm withdrew from the case. Now the defendant has asked Cheryl and her firm to take over the defense. Which of the following is most nearly correct? A. If Cheryl and her firm agree to represent the defendant, then Dennis and his firm will be subject to discipline if they do not seek their client's permission to withdraw. B. Cheryl and her firm will be subject to discipline if she agrees to represent the defendant, since to do so would create an appearance of impropriety. C. If the respective clients consent after full disclosure of the situation, then Cheryl and Dennis may participate on opposite sides of the case. D. Cheryl and Dennis may participate on opposite sides of the case, since the mere fact that they are married creates neither an actual nor an apparent conflict of interest.

1. D. It is proper, indeed routine, for a lawyer to talk with a witness about his testimony before the witness testifies. It is also proper for a lawyer to use documents or other items to try to refresh a witness's memory of a once-known but now forgotten fact. In this process, the lawyer must bear in mind that the proper object is to "extract the facts from the witness, not to put them into him; to learn what the witness does know, not to teach him what he ought to know." [In re Eldridge, 82 NY. 161, 171 (1880); see also RESTATEMENT (THIRD) OF THE LAW GOVERNING LAWYERS § 116(1)(2000).] Items I and II are incorrect because the question does not indicate that Westerman exceeded these bounds in meeting with the investigator. Item III is correct—this kind of sarcastic, rebellious remark in the judge's presence in open court constitutes direct contempt of court, making the lawyer subject to litigation sanctions. It could also result in professional discipline under ABA Model Rule 3.5(d). Item IV is correct; a lawyer is subject to litigation sanctions for intentionally violating an established rule of evidence law; here production of the notes was clearly required under Federal Rule of Evidence 612. Professional discipline would also be appropriate under ABA Model Rule 3.4(c). Item V is correct; the judge ordered Westerman to bring the notes "the next morning," and Westerman intentionally failed to do so. His conduct constitutes another direct contempt of court.

1. At the trial of a routine civil case in a United States District Court, defense lawyer Westerman presented the testimony of an insurance company investigator. On cross examination, plaintiff's lawyer established that on the day before the trial began, the investigator spent three hours in Westerman's office going over his testimony. On that occasion, Westerman showed the investigator some handwritten notes from the insurance company files, in an effort to refresh the investigator's recollection of some important dates. Plaintiff's counsel asked to have the notes brought to court the next morning; after hearing oral argument on the point, the judge ordered Westerman to bring them the next morning. Westerman responded: "I'll bring them, judge, on the next cold day in Hell." The judge looked startled but chose to overlook the remark. Westerman intentionally failed to bring the notes to court the following day. Which of the following are correct? I. Westerman is subject to litigation sanctions for discussing the investigator's testimony with him before the trial. II. Westerman is subject to litigation sanctions for using the notes to refresh the investigator's memory of dates. III. Westerman is subject to litigation sanctions for his rude remark to the judge. IV. Westerman is subject to litigation sanctions for intentionally violating the Federal Rules of Evidence. V. Westerman is subject to litigation sanctions for intentionally violating the judge's order. A. All of the above. B. II and V only. C. I, II, III, and IV only. D. III, IV, and V only.

1. A. ABA Model Rule 5.5(a) prohibits Alford from practicing in a state where he is not admitted to practice. He can avoid this prescription if he is admitted pro hac vice to defend Clara in the State B case. [See ABA Model Rule 5.5(c)(2) and Comments [9]—-[11].] Answer B is not correct because requiring admission to practice does not discriminate against non-residents—neither residents nor non-residents can practice law without being admitted. Answers C and D are not correct because if Alford is admitted pro hac vice he may represent Clara in the case even though it involves a State B business and the interpretation of a State B statute.

1. Attorney Alford is admitted to practice before the highest court of State A, but not in State B. Client Clara lives in State A, but she runs a business in State B. She asks Alford to defend her in a lawsuit pending in a trial court of State B. The suit involves the proper interpretation of a State B business tax statute. Would it be proper for Alford to represent Clara? A. Yes, if the State B court admits him pro hac vice, that is, for the sole purpose of litigating this case. B. Yes, because State B cannot constitutionally discriminate against non-resident attorneys. C. No, because Clara's business is conducted in State B, and he is not admitted to practice in State B. D. No, because the suit involves the interpretation of a State B statute, and he is not admitted to practice in State B.

1. D. Item III is correct. ABA Model Rule 1.5(d)(2) makes Lenox subject to discipline for using a contingent fee in a criminal case. Item I is also correct; Lenox should not have taken this case on contingent fee, but since he did, he should at least have put the contingent fee agreement in writing. [ABA Model Rule 1.5(c).] Items II and IV are both correct. ABA Model Rule 1.8(e) permits Lenox to advance the litigation expenses, and Denmon's promise to pay back the advance is proper under ABA Model Rule 1.8(e).

1. Criminal defense lawyer Lenox agreed to represent defendant Denmon at Denmon's trial for arson. Lenox and Denmon orally agreed on the following attorney fee arrangement. If Denmon were acquitted, the, fee would be $25,000. If Denmon were convicted of any lesser included offense, the fee would be $5,000. If Denmon were convicted of arson. the fee would be $500. Lenox further agreed to advance all litigation expenses, subject to Denmon,S promise to repay Lenox whatever the outcome of the case. Which of the following statements are correct? I. Lenox is subject to discipline for not putting the fee agreement in writing. II. It was proper for Lenox to agree to advance the litigation expenses. III. Lenox is subject to discipline for charging a contingent fee in a criminal case. IV. It was proper for Lenox to require Denmon to repay the advanced litigation expenses whatever the outcome of the case. A. Only I, II, and IV are correct. B. Only I and III are correct. C. Only II and IV are correct. D. All of the statements are correct.

1. C. The inability to work effectively with the client's chosen co— counsel—coupled with harm to the client's interests—have long been recognized as a sufficient reason for permissive withdrawal. [See ABA Model Rule 1.16(b)(6) (client has made representation unreasonably difficult); see also ABA Code DR 2—110(C)(3) (specific mention of inability to work with co-counsel).] Answer A is not as good as C, because it does not take into account Snyder's duty to give advance notice and to take other steps to avoid prejudice to the client. [ABA Model Rule 1.16(d).] Answer B is incorrect because the lawyer cannot "instruct" the client about matters like this—the choice of co-counsel is for the client, not the lawyer. Answer D suffers the same defect; further, it might be regarded as an improper interference with the contractual relations between Slick and the client.

1. For many years, lawyer Snyder has represented a profeSSional football team, the Raptors, in business law matters. On the team's behalf, Snyder has filed a breach of contract case against the City Board of Commissioners concerning the stadium that the city leases to the Raptors. Snyder is counsel of record in the suit, and he has conducted all of the discovery for the Raptors. The trial date is fast approaching, and the Raptors' owners have retained a famous trial lawyer, Marvin Slick, to serve as Snyder's co-counsel and to do the actual trial work. Although Snyder envies Slick's win-loss record, he regards Slick as little more than a highly-educated con artist with whom he cannot possibly work. Which of the following may Snyder do? A. Immediately seek the court's permission to withdraw from the case. B. Promptly instruct the team owners to terminate their arrangement with Slick. C. Ask the team owners to consent to his withdrawal, if he believes that is in their best interests. D. Advise Slick to withdraw, if Snyder believes that is in the best interests of the team owners.

1. C. The attorne_\~'-client. privilege covers the information in item I. The tax accountant was simply acting as a conduit to help communicate information from the client to the attorney, and the tax accountant's role was to help further the att.orne.v_'-client relationship. The information is also covered by the attorney's ethical duty to preserve confidential information. [See ABA Model Rule 1.6.] The items of information in items II. III. and IV are likewise covered by the ethical duty, but they are not covered by the attorney-client privilege because the attorney did not obtain the information through confidential communications with the client. In item II. the information came from public land records. In item III. it came from a third party taxi driver. In item IV, it came from a public newspaper.

1. In which of the following situations would the information received by the attorney be covered by both the attorney-client privilege and the ethical duty to preserve the client's confidential information? I. Lawyer L is defending client C in a tax fraud case. With C's consent, L hires a tax accountant to examine C's records, to talk with C, and to prepare some worksheets for L to use in defending the case. The accountant turns the worksheets over to L. II. L is representing C in a boundary line dispute with C's neighbor. When combing through the county land records, L discovers that C's grantor apparently had no legal title to the land he purported to grant to C. III. L is defending C in a first degree murder case. In the course of her investigation, L talks to a taxi driver who tells L that he remembers that on the night in question C rode in his taxi to an address near the scene of the murder. IV. L represents C in an action for breach of an oral contract. When preparing the case for trial, L stumbles across an old newspaper clipping, reporting C's conviction of a felony in a distant state 15 years ago. A. All of the above. B. I, III, and IV only. C. I only. D. III only.

1. C. If Arnold acquires 4% of the capital stock of the new corporation, he will in essence. be entering into a business transaction with his ch'ents. Ivan and Gene. [See ABA" Formal Op. 00—4-18 (2000) (discussing Circumstances in which a lawyer can acquire stock of a start- up client in lieu of a fee).] Therefore. Arinold Will" have to compl}~ with ABA Model Rule 1.8(a), which requires all of the conditions listed in C. except for the first one. The transaction must. also satisfv this first condition because Arnold is accepting the stock in' lieu of a fee. and an attornexfs fee must not be unreasonable in amount. That would depend on how much the stock was worth, viewed as of the time of the transaction. not through the lens of hindsight. Answer A is wrong because it nn'sstates the rule. The rule in' question is ABA Model Rule 1.8(1'). which speaks of the "subject matter of litigation," not the "subject of representation There is no litigation in this problem. An'swer B is wrong because it also misstates the rule. The rule in question is ABA Model Rule 1.8(a). which allows a lawyer to enter into a business transaction with a client if the specified conditions are satisfied. Answer D is wrong because it does not st-itisfy all the conditions of ABA Model Rule 1.8(a). and it incorporates an I'lliag'lahal'y rule about not voting the stock or trxfing to influence governance of the corporation. Note, however. that if the lawyer's acquisition of stock might affect control of the corporation, that is one of the potential conflicts that the lawyer should fully disclose to the clients under Rule 1.8(a). [Sec ABA Formal Op. 00—418.]

1. Inventor Ivan and marketing genius Gene want to form a new corporation to market Ivan's amazing new design for motion picture projectors. They want to hire attorney Arnold to help them to do the necessary legal work and to help them find venture capital. Because they have almost no hard cash at present, they have asked Arnold to do this work for them in exchange for 4% of the capital stock of the new corporation. The remaining 96% will be divided equally between Ivan and Gene and their respective families. May Arnold agree to their proposal? A. No, because a lawyer must not acquire a personal interest in the subject of the representation. B. No, because a lawyer must not enter into a business transaction with clients. C. Yes, but only if the 4% would not make the fee unreasonably high, and the transaction would be fair to the clients, and the terms are fully disclosed to the clients in an understandable writing, and the clients are advised in writing of the desirability of seeking outside counsel and given a chance to consult such outside counsel, and the clients consent in writing. D. Yes, but only if Ivan and Gene give their informed consent and Arnold promises that he will never vote his stock or otherwise attempt to influence the governance of the corporation.

1. A. ABA Model Rule 7.2(b) prohibits a lawyer from giving "anything of value" to a person for "recommending the lawyer's services." The items of gossip Philos feeds to Norris are items of value to Norris, and Norris's favorable comments about Philos are in the nature of recommendations of his services. Answer B is wrong because there is no such disciplinary rule. Answer C is wrong because Philos is subject to discipline on the ground stated in answer A. Answer D is wrong because this kind of verbal conduct is not protected speech.

1. Mark Norris is a newscaster for the local television station. Eveyrv weeknight. following the evening news, he presents a ten-minute segment entitled "This Funny Town." It is patterned on an old-fashioned newspaper gossip column. Most of it concerns the private lives and peccadilloes of the prominent and would-be prominent citizens of the community. Judges and lawyers are among Mr. Norris's favorite subjects. He and attorney Philos have arrived at a tacit arrangement. Whenever Philos hears a piece of juicy gossip about a local judge or lawyer, he passes it along to Norris. In return, Norris frequently recommends Philos's legal services in his broadcasts. For example, Norris calls Phil'os "a fearless courtroom ace," or he states opinions such as: "if you want to win your case, hire Philos." Is Philos subject to discipline? A. Yes, because Philos is providing something for recommending his services. B. Yes, because a lawyer can be disciplined members of the legal profession. of value to Norris for demeaning other C. No, unless he gives false or privileged information to Norris. D. No, because Philos's conduct is a protected under the First and Fourteenth Amendments.

1. D. ABA Model Rule 1.3 provides that a lawyer must "act with reasonable diligence and promptness in representing a client." Further, ABA Model Rule 1.4 requires a lawyer to keep the client "reasonably informed about the status of the matter," and ABA Model Rule 8.4(c) forbids lying and other dishonest conduct. Answer A is not correct. Even if Acevedo would have been able to do the work on time, he is subject to discipline for lying to Catlin about his progress on the matter. Answer B is not correct. Acevedo is subject to discipline even though Catlin was able to find another lawyer who could get the complaint filed in time. Answer C is not correct. Since Catlin was not injured. by the delay, Acevedo is not liable for malpractice.

1. On June lst, client Catlin hired attorney Acevedo to sue defendant Degan for securities fraud. Catlin and Acevedo realized that the complaint would have to be filed by September 15th to be within the statute of limitations. Acevedo was very busy with other matte-rs. Starting in mid- August, Catlin telephoned Acevedo every few days to see what progress Acevedo was making. Acevedo repeatedly assured Catlin that he was assembling the facts and preparing preliminary drafts of the complaint. but in truth Acevedo was doing nothing on the case. On September 10th, Catlin learned from Acevedo's secretary that Acevedo had still not started to work on the case. At that point, Catlin fired Acevedo and hired a different lawyer who was able to get the complaint on file by September 15th. Although Acevedo did not charge Catlin any fee, Catlin reported the matter to the state bar. Which of the following is most nearly correct? A. If Acevedo would have been able to complete the necessary work by September 15th, his conduct was proper. B. Since Catlin suffered no damage due to Acevedo's delay, Acevedo's conduct was proper. C. Even though Catlin suffered no damage due to Acevedo's delay, Acevedo is subject to civil liability for malpractice. D. Acevedo is subject to discipline for neglecting Catlin's case and for lying to Catlin about the status of the matter.

1. B. Answer A is incorrect. The First Amendment does not protect all communications. The associates' comments reflect bias, and therefore Answer D is incorrect. Answer B is correct rather than Answer C because there is no provision within the ABA Model Rules that would subject the associates to discipline.

1. Several associates from the mid-size law firm of Harrison & Malloy went out together for dinner at a local restaurant. During the course of the dinner, two members of the group confided that they were quite put off by the behavior of a male colleague, which they characterized as effeminate and as indicative of homosexuality. A. The First Amendment protects the associates' comments. B. Although the associates' comments reflect bias, under these circumstances the Model Rules contain no prohibition. C. The associates' comments reflect bias and therefore the associates are subject to discipline D. The associate's comments do not reflect bias because they are entitled to their personal opinions.

1. B. The relevant provision is ABA Model Rule 3.3(a)(2), concerning a lawyer's duty to alert the court to adverse legal authorities. In Item I, the United States Supreme Court case is adverse only by analogy between the State X statute and the federal Lanham Act. Further, the United States Supreme Court's interpretation of federal law is not controlling on a State X judge who is applying State X law. For the same reasons, the United States Court of Appeals case in Item II need not be disclosed. The State Y case in Item III need not be disclosed because State Y law is not controlling in a case governed by State X law. The State X case in Item IV need not be disclosed because it is adverse only by analogy between trespass to real property and infringement of a trademark. There may be sound tactical reasons for counsel for Noxatox to call all of these cases to the court's attention, but the rules do not compel it.

1. State X and State Y each have state trademark registration statutes that are substantially similar in purpose and wording to the Lanham Act (the federal trademark registration statute). For many years, Daisy Dairy has used the mark "Daisy" on dairy products it sells in State X, and it has registered the mark under the State X statute. Recently Noxatox Chemical began using the "Daisy" mark on cockroach poison it sells in State X. Daisy Dairy sued Noxatox under State X law in a State X court for intentional infringement of the "Daisy" mark. The complaint asks for an injunction, for an award of the profits made by Noxatox, and for money damages. Noxatox moved for summary judgment on the grounds that dairy products and cockroach poison do not compete with each other, that no sensible consumer could be deceived by the use of the same mark on such widely different goods, and that Daisy Dairy could not possibly have suffered monetary injury. The trial judge who will hear the motion is not well versed in trademark law, and the lawyer for Daisy Dairy failed to discover several pertinent court dec1s1'on3 Which of the following decisions must the lawyer for Noxatox call to the judge's attention? I. A United States Supreme Court decision which holds that the Lanham Act authorizes an injunction to stop intentional infringement, even where the defendant's goods do not compete with the plaintiff"s goods. 11. A decision of the United States Court of Appeals for the circuit that includes State X and State Y, holding that an injunction can be issued under the Lanham Act where the nature of the defendant's goods could cast a distasteful or odious image on the plaintiff's goods. III. A decision of the Supreme Court of State Y which holds that the State Y registration statute authorizes an accounting of the defendant's profits in a case of intentional infringement, even Where the plaintlff cannot prove monetary injury. IV. A decision of the Supreme Court of State X which holds that in actions for intentional trespass to real property, State X trial judges have the power of courts of equity to fashion equitable remedies, even where the plaintiff cannot prove monetary injury. A. All of the above. B. None of the above. C. I, II, and IV only. D. I and IV only.

2. C. The judge's reference to "you people" indicates bias. Canon 3(B)(5) of the CJC provides that judges "shall not, in the performance of judicial duties, by words or conduct, manifest bias or prejudice" on the basis of disability. Accordingly, Answers A and D are incorrect. B is also incorrect; the judge's response was inappropriate because it reflected bias, not because the judge was required to accommodate Matthew's specific request. Although the judge should have sought some means to preserve the confidentiality of the information that Matthew sought to communicate, the judge was not necessarily required to follow Matthew's particular suggestion.

2, Matthew Fontaine is confined to a wheelchair as the result of an automobile accident ten years ago. He practices law as a sole practitioner. Although Matthew generally handles non-litigation matters, he was persuaded to accept a litigation case to help "a friend of a friend." In the course of trial, Matthew asked to approach the bench in order to discuss a matter of some confidence. Due to the height of the bench, and Matthew's inability to stand, Matthew asked the judge to come around to the front of the bench in order to maintain the confidentiality of the discussion. The judge refused. and stated. "This is my court. If you people insist on practicing law. you have to deal with life's realities instead of asking for special favors." A. The judge's response was appropriate. B. The judge's response was inappropriate because he was required to accommodate Matthew's request. C. The judge's response reflects bias and he is potentially subject to discipline. D. The judge's response does not reflect bias but he should have sought some means of preserving confidentiality.

2. D. Item I is correct because the problem specifies that the local Rules of Court require court permission before an attorney withdraws from a litigated case, and the problem further states that courts have statutory authority to enforce the Rules of Court with litigation sanctions. Item II is incorrect because Arbuckle has ample grounds for withdrawal: Clauzoffs refusal to co-operate in discovery and his refusal to pay Arbuckle's fee bill are each sufficient. Item III is correct because the problem states that State A does not recognize attorney retaining liens. Item IV is correct because when a lawyer withdraws, she must refund the unspent part of the expense advance. [ABA Model Rule 1.16(d)(lawyer must return property to which client is entitled); see also RESTATEMENT [THIRD] OF THE LAW GOVERNING LAWYERS § 33, comment e, and § 45(1) (lawyer must surrender funds belonging to client).]

2. Attorney Arbuckle is admitted to practice in State A. The State A Rules of Court require court permission before an attorney can withdraw from a pending case. The courts of State A have statutory authority to impose litigation sanctions on lawyers who Violate the Rules of Court. State A does not recognize attorney retaining liens on client's funds, property, or litigation files (that is, an attorney cannot keep these items to secure payment of his or her fee). Arbuckle agreed to defend Clauzoff in a civil action for theft of the plaintiff's trade secrets. Clauzoff agreed to pay Arbuckle $100 per hour. and he gave Arbuckle a $10,000 advance for litigation expenses. Three times, the plaintiff scheduled the taking of Clauzoff's deposition, and all three times Clauzoff failed to show up. Further, despite repeated promises, Clauzoff failed to send Arbuckle some documents that Arbuckle needed in order to draft responses to the plaintiff's interrogatories. After Arbuckle put in 40 hours on the case, he billed Clauzoff $4,000, but Clauzoff refused to pay the fee bill. Finally, Arbuckle decided to have nothing further to do with the case; when plaintiff's counsel telephoned, Arbuckle told her that he had resigned as the lawyer for Clauzoff. Clauzoff asked Arbuckle to hand over the litigation files and to refund the unspent part of the $10,000 expense advance, but Arbuckle refused both requests. Which of the following statements are correct? I. Arbuckle is subject to litigation sanctions for stepping out of the case without the court's consent. II. Arbuckle is subject to discipline for withdrawing from the case without adequate grounds. III. Arbuckle is subject to discipline for refusing to hand over the litigation files to Clauzoff. IV. Arbuckle is subject to discipline for refusing to refund the unspent part of the expense advance. A. Alloftheabove. B. I and IV only. C. II, III, and IV only. D. I, III, and IV only.

2. C. This is a debatable legal ethics issue. Black is the supervising attorney. and it is Black's client. Arlington is simply assisting Black, so Black should make the final judgments on debatable ethics issues. [ABA Model Rule 5.2.] Answer C is preferable to answer B because B is overbroad. Where the supervisor and the subordinate are faced with a debatable ethics issue, the supervisor's judgment should prevail. But, if the question were not debatable, the subordinate would be subject to discipline, even though he was carrying out the directions of the supervisor. [ABA Model Rule 5.2(a) and (b); see generally Hazard & Hodes § 43.5.]

2. Attorney Arlington is a young associate in the firm of Smith & Black. He is assisting senior partner Black in the discovery phase of a case in which the court has ordered Black's client to produce certain documents. Black asked Arlington to study the court order, to review several boxes of documents sent over by the client, and to decide which documents must be produced. Arlington did the work and presented his conclusions to Black. Black and Arlington disagree about one group of documents. Black maintains that the court order does not require them to be produced, but Arlington insists that a fair reading of the court order does require them to be produced. The two attorneys agree that the question is a close one, but each is convinced that the other is incorrect. Which of the following is most nearly correct? A. If Arlington gives in to Black's point of View, Arlington will be subject to discipline, since an attorney must not hold back what a court has ordered to be produced. B. Since a subordinate attorney cannot be held accountable for following the directions of a supervising attorney, Arlington must accede to Black's point of view. C. Since the point is a debatable one, Arlington may accede to Black's point of View. D. Since an attorney is required to follow his own. independent judgment in handling a client's matter, Arlington must either insist that the documents be produced or else decline to work further on the case.

2. B. Martha's use of a contingent fee agreement in this divorce case, and her failure to discuss alternative fee arrangements with Kimberly, make this a good candidate for a partial fee forfeiture under the Restatement. [See RESTATEMENT (THIRD) OF THE LAW GOVERNING LAWYERS § 37 (2000).] Martha ignored the advice of ABA Formal Op. 94— 389 to fully inform the client about other ways to pay the fee before entering into a contingent fee agreement, and then she tried to exact $2.5 million from Kimberly. That strikes us as a "clear and serious violation of a duty to a client" under §37. Item C is incorrect because ABA Model Rule 1.5(d)(1) prohibits a contingent fee in a domestic relations case where the contingency is the amount of a property settlement. Item A is incorrect because a competent family lawyer should have foreseen a large property settlement on the facts of this case; Martha ran little risk of not getting paid, and that ought to influence the court's determination of whether $2.5 million is reasonable. [See ABA Formal Op. 94—389.] Item D is incorrect because it is overbroad. The Comment to ABA Model Rule 1.5 urges lawyers to use arbitration or mediation to resolve fee disputes, but there is no prohibition on suing to collect a legal fee when necessary.

2. Attorney Kimberly represented client Marsha in a divorce proceeding in a non-community property jurisdiction that has a no-fault divorce law. Marsha was married for 25 years, and during the marriage her husband became a very wealthy business executive. The two children of the marriage grew up and left home. Marsha had a savings account of her own, but she was not wealthy, and she had no marketable job skills. Kimberly convinced Marsha to sign a contingent fee agreement, in which Kimberly's fee would be 25% of whatever property settlement Marsha would get in the divorce decree. The divorce court awarded Marsha a $10 million dollar property settlement. Marsha refused to pay Kimberly the $2.5 million fee due under the fee agreement, saying that it was unreasonably high. After trying without success to settle the fee dispute amicably, Kimberly sued Marsha to collect the fee. May the court award Kimberly less than $2.5 million? A. No, because Kimberly took the risk of not being paid anything when she took the case on contingency, and she is entitled to be compensated for that risk. B. Yes, because Kimberly acted improperly in using a contingent fee agreement in a divorce case, where the amount of the fee was controlled by the amount of the property settlement. C. No, because a contingent fee agreement is permissible in a domestic relations case, except where the contingency is the granting of a divorce. D. Yes, because it is unethical for a lawyer to sue her own client to collect a fee.

2. C. Items I and II are not correct. Alder provided no legal services in connection with Christenson's proposed beachfront development; Alder was Christenson's divorce lawyer. Accordingly, this situation does not come within any of the exceptions to the duty of confidentiality under ABA Model Rule 1.6. and Item IV is correct. Item III is correct; surely Al'der may volunteer his advice in this context, even though Christenson will probably ignore it.

2. Client Christenson asked attorney Alder to prepare some legal papers in connection with Christenson's dissolution of marriage proceeding. In the course of conversation, Alder learned that Christenson intended to develop some beachfront property into condominiums. State law requires the filing of certain environmental impact statements with the State Commissioner of Real Estate and Development as a prerequisite to any development efforts, including advertising and zoning variances. Later Alder learned that Christenson was proceeding with the project and had not yet filed the required statements. Which of the following items are correct? I. Alder must contact the State Commissioner of Real Estate and Development and reveal Christenson's intentions. II. Alder may contact the State Commissioner of Real Estate and Development and reveal Christenson's intentions. III. Alder may contact Christenson and urge him to take appropriate steps to rectify his wrong. IV. It would be proper for Alder not to tell any outsider about his communications with Christensen. A. I, III, and IV only. B. II, III, and IV only. C. III and IV only. D. IV only.

2. B. Taking Norman's name off the pleading does not solve the conflict of interest. Norman remains the outside general counsel for North American, and North American may be liable on any judgment rendered against the joint venture in the suit brought by Eratec's French subsidiary. The proper course of action here would have been: (a) for Enid and Norman to decide whether they could effectively serve their respective clients in this situation; (b) if they concluded that they could, they should have made full disclosure of the problem to both sets of clients (not just to North American); and (c) they should have obtained the written consent of both sets of clients. [See ABA Model Rule 1.7.]

2. Law partners Norman and Enid are too busy to spend much time discussing their legal work with each other. For many years, Enid's major client has been Eratec Corporation, a diversified electronics firm with worldwide operations. Most of Norman's time is devoted to his work as outside general counsel for North American Industries, Inc. It is a diversified manufacturing company with operations in Canada, the United States, and Mexico. Enid filed a law suit in the United States on behalf of a French subsidiary of Eratec. The defendant was a Canadian joint venture. When Enid received the answer to the complaint, he was shocked to discover that North American was one of the three joint venturers, and that Norman was listed on the caption of the answer as "Of Counsel" to the joint venture. Immediately after Enid called this to Norman's attention, Norman explained the situation to North American and to the lead counsel for the joint venture. Both readily consented to the removal of his name from the pleading. Enid then continued in the case as counsel for the plaintiff, Eratec's French subsidiary. Which of the following is most nearly correct? A. Enid and Norman handled the matter in a proper manner, since Norman had his name removed from the pleading promptly after the conflict of interest was discovered. B. Enid is subject to discipline, even though Norman's name was promptly removed from the pleading. C. Neither Enid nor Norman is subject to discipline, since the conflict of interest was unintentional and was remedied as soon as it was discovered. D. Enid handled the matter in a proper manner, but Norman is subject to discipline for failing to discover the conflict of interest at the outset.

2. B. Lenschell's ch'ent is Tina, not Timothy. no matter who may be paying fine or the legal fee. Therefore. Lenschell must not allow Timothy to orchestrate the case. [See ABA Model Rule 1.7(a): ll'oo'd 1'. Georgia. 450 US 261, 101 S.Ct. 1097, 67 L.Ed.2d 220 (1981) (pornography shop employee defended by lawyer who was paid by shop owner)]. Whether to plead guilty or not guilty is for Tina to decide. [ABA Model Rule 1.2(aH Thus, answers A and D are incorrect. Answer B is prefera-ible to answer C. Lenschell has gotten this far with the case. and time is short since the case is about to be called on the criminal calendar. Assuming that Tina wants Lenschell to advise her about the plea, he should do so. The question does not provide enough facts to decide whether it would or would not be appropriate for Lenschell to continue representing Tina at subsequent stages of the case. Note that ABA Model Rule 1.8m prohibits

2. Lawyer Lenschell has recently opened his new law office. Timothy came to Lenschell's office and introduced himself as the "boyfriend" of Tina, a young woman who was just arrested on a prostitution charge. Timothy retained Len'schell to represent Tina and paid him an appropriate'fee in advance. Timothy, who seemed to know a great deal about the law pertaining to prostitution despite being a layperson, explained to Lenschell that in prostitution cases in this district, a guilty plea usually results in a $500 fine. but no jail sentence. But if the defendant pleads not guilty, goes to trial, and is found guilty. the. judge usually imposes a jail sentence. Timothy further explained that Tina did not want to go to jail, that he would pay her fine for her, and that Lenschell should therefore advise her to plead guilty. Lenschell met Tina for the first time at the courthouse, shortly before her case was to be called on the criminal calendar for entry of her plea. In their hurried conference, Tina told Lenschell that Timothy was her pimp, not her "boyfriend." Further, she said that she wanted to escape from Timothy and from her life as a prostitute, and that she wanted to plead not guilty, thus risking a jail sentence, rather than become further indebted to Timothy. What is the proper course of conduct for Lenschell to follow in this situation? A. To adhere to the instructions given by Timothy, and to advise Tina to plead guilty. B. To give Tina whatever legal assistance she needs in entering her plea of not guilty. C. To withdraw from the matter promptly, without advising Tina one way or the other on what plea to enter. D. To telephone Timothy and ask for further instructions in light of Tina's unwillingness to plead guilty.

2. D. ABA Model Rule 4.2 prohibits a lawyer from communicating about the subject of the representation with a person who is represented by a lawyer, without first getting the consent of that lawyer. Comment 7 to ABA Model Rule 4.2 explains that when the represented party is a legal entity (such as the church corporation in this question), a lawyer must get the consent of the entity's lawyer before talking directly with three classes of persons: (1) persons who "supervise[ ], direct[ ], or regularly consult[] with the organization's lawyer concerning the matter"; (2) persons who have the "authority to obligate the organization with respect to the matter"; and (3) persons whose "act or omission in connection with the matter may be imputed to the organization for purposes of civil or criminal liability." [Cf. Upjohn Co. v. United States, 449 US. 383, 101 S.Ct. 677, 66 L.Ed.2d 584 (1981) (defining scope of attorney-client privilege as to communications between corporate employees and attorneys for the corporation).] Even if the bookkeeper does not fall within class (1) or class (2), we believe that she falls within class (3). Thus, Lexington should have gotten the consent of counsel for the church corporation before talking with her. [See also Hazard & Hodes § 38.6.]

2. Lawyer Lexington represents the plaintiffs in a civil action. His clients are three members of the congregation of All Souls' Divine Missionary Church, suing on behalf of themselves and others similarly situated. The defendants are All Souls' Divine Missionary Church, Inc., (a corporation) and Pastor Dorset, the spiritual leader of the church and president of the church corporation. Pastor Dorset and the church corporation are represented by separate defense lawyers. The complaint alleges that Pastor Dorset misappropriated large amounts of church money, and that the Board of Elders, acting as corporate directors, knew about it and failed to stop him. In the early discovery phase of the case, lawyer Lexington conducted a lengthy, private interview with the church bookkeeper, an employee of the church corporation; she brought the church books with her to the interview, and she and Lexington went over them in great detail. Lexington did this without the knowledge or consent of either defense lawyer. Which of the following is most nearly correct? A. Lexington's conduct was proper, since the bookkeeper was not a party to the lawsuit. B. Lexington's conduct was proper, since the bookkeeper was neither an officer nor a high-ranking employee of the church corporation. C. Lexington is subject to discipline; he should have obtained the consent of both defense lawyers. D. Lexington is subject to discipline; he should have obtained the consent of the church corporation's defense lawyer.

B. If Linda tells the bar of State B that her cousin is fit to practice law, when in fact she believes him to be thoroughly dishonest, she would knowingly be making a false statement of material fact in violation of ABA Model Rule 8.1(a). Answers C and D are not correct for the same reason. Answer A is not correct—Linda's lack of membership in the bar of State B is beside the point. State A could discipline her. for lying to the bar of State B. [See ABA Model Rule 8.5.]

2. Lawyer Linda is admitted to practice in State A, but not in State B. Her cousin asks her to write a letter recommending him for admission to practice law in State B. Linda knows that her cousin is educationally well- qualified to be a lawyer, but she regards him as thoroughly dishonest. May Linda write a letter stating that her cousin is fit to practice law? A. No, because Linda is not a member of the bar of State B. B. No, because Linda would be making a false statement of a material fact. C. Yes, because her belief about her cousin's lack of honesty is merely her own opinion. D. Yes, because the bar of State B will decide for itself whether her cousin is a person of good moral character.

2. C. Item I is correct. The question states that any reasonably competent general practitioner would have discovered the more favorable law under the Lanham Act. Lloyd failed to discover it, and she is thus liable for the injury Cress suffered due to her negligence. Item II is correct. As the partner in charge of this case, Ames himself was probably negligent for taking the case to trial on state law theories only. Even if that were not true, partner Amos is liable for Lloyd's negligence under ordinary principles of respondeat superior. Item 111 is correct. As a partner in the firm, Baker is liable for the negligent acts of Lloyd and Ames. Item IV is not correct for the reasons explained above.

2. Lawyer Lloyd was an associate attorney employed by the law firm of Ames & Baker. The firm is an ordinary partnership, not a limited liability entity. Client Cress hired Ames to sue one of his competitors for false advertising. Ames assigned Lloyd to do the necessary research and draft the complaint. Lloyd confined her research to state law. Any reasonably competent general practitioner would have discovered a more favorable body of parallel federal law under Section 43(a) of the Lanham Act. Ames eventually brought the case to trial on state law theories only, and Cress lost. Had the case been tried under the Lanham Act, Cress would have won a large judgment. Which of the following propositions are correct? I. Lloyd is subject to civil liability for malpractice. II. If Lloyd is subject to civil liability for malpractice, then so is Ames. III. If Lloyd and Ames are subject to civil liability for malpractice, then so is Baker. IV. None of the three lawyers is subject to civil liability for malpractice. A. Only IV is correct. B. Only I is correct. C. Only I, II, and III are correct. D. Only I and II are correct.

2. C. ABA Model Rule 3.3(d) requires a lawyer in an ex parte proceeding to disclose all of the relevant facts known to the lawyer, even the adverse facts. Comment 14 to Model Rule 3.3 explains why.

2. Lawyer Penny represents client Paul in a family law matter. When Paul and Donna were divorced, the court gave Paul custody of their infant son and gave Donna "reasonable" visiting rights. Paul is a busy accountant and often stays late at his office. While Paul is working, the baby stays at a baby sitter's house. Donna has started making unannounced visits to the baby sitter's house on the evenings when Paul works late. Paul believes Donna may try to kidnap the baby and disappear. Paul asks Penny to apply immediately for a temporary restraining order that forbids Donna from going near the sitter's house. The court rules of this jurisdiction provide that a temporary restraining order can be granted in an ex parte proceeding, without giving the adversary any notice or chance to be heard. Penny plans to use this ex parte rule. Penny has an affidavit from the sitter stating that when Donna makes her surprise Visits, the baby cries and refuses to eat or sleep for hours thereafter. Just as Penny is leaving her office to go to the judge's chambers, her investigator arrives with three additional pieces 0f information. First, when Paul works late, the sitter sometimes leaves a ten— year-old neighbor girl in charge of the baby while the sitter grocery shops and runs errands. Second, Donna's unannounced visits are motivated by her concern for the baby's safety. Third, when Paul works late, Donna could Conveniently keep the baby at her house until Paul is through at the office. which of the followmg is most nearly correct? A- Penny must present the judge with only those facts that favor Paul's position. B. Penny must present the judge with only those facts that favor Paul's position, but she must respond candidly if the judge specifically asks for information that is adverse to Paul's position. C. Penny must present the judge with all the relevant facts, even those that are adverse to Paul's position. D. Penny must call Paul and tell him that she is withdrawing the application for a temporary restraining order.

2. D. The relevant authority is ABA Model Rule 7.5 and its comments. Item I is correct; the name of a dead partner may be retained by a successor firm. Item II is incorrect; the firm name makes Trimble appear to be a partner when he is in fact an associate. Item III is incorrect; Snod's name should have been removed when he ceased the regular practice of law to enter government service. Item IV is incorrect; the sign on the door makes Tremble and Gangler appear to be partners when in fact they are not.

2. Three years ago, attorneys Hooten and Snod formed a law partnership called Hooten & Snod. A year later, Hooten died, and Snod continued practicing under the former firm name. Then Snod hired a salaried associate, attorney Tremble, and, the firm name was changed to Hooten, Snod & Tremble. The following year, Snod left law practice to become a commissioner on the Federal Trade Commission. Tremble took over the practice and continued to use the same name. Later, because he had more space in the office than he needed, he entered into a space-sharing agreement with attorney Gangler. The sign on the door now reads Tremble & Gangler, Attorneys at Law. Which of the following are correct? I. After Hooten died, it was proper for Snod to continue using the firm name Hooten & Snod. II. When Snod hired Tremble, it was proper to change the firm name to Hooten, Snod & Tremble. 111. After Snod joined the FTC, it was proper for Tremble to continue using Snod as part of the firm name. IV. The present sign on the door is proper. A. All of the above. B. I, II, and III only. C. I and II only. D. I only.

3. A. ABA Model Rule 1.17 allows the sale of a law practice, subject to certain conditions. Rule 1.17 specifically permits the good will of a law practice to be sold, contrary to the prior law in most jurisdictions. The $500 monthly retirement benefit does not offend any provision of the ABA Model Rules. Answer B is wrong because there is no such "reasonableness" requirement. Answer C is wrong because it is contrary to the opening clause of ABA Model Rule 1.17. Answer D is wrong because there is no requirement that the buyer and seller of a practice have been previously associated in practice.

3. After 45 years of solo practice in the small town of Willow (,a'reek. lawyer Lumire decided to sell his law practice and to retire. Lumiere advertised the practice for sale in the classified pages of the local bar journal, and in due course he found a buyer, an enthusiastic young attorney named Ames. The sales contract between Lumire and Ames provides that Ames will pay Lumire $65,000 for the small wood building that houses Lumire's office; $8,000 for the furniture, law books, office machines, and related items of personal property; and $10,000 for the good will of the practice. It further provides that Ames will pay Lumire $500 per month as a retirement benefit during Lumire's lifetime or until Lumire returns to law practice in Willow Creek. 13 the sales contract proper? A. Yes, even though it provides for the $500 monthly retirement benefit. B. Yes, provided that the $500 monthly retirement benefit is reasonable in light of the good will value of the practice. C. No, because the good will of a law practice cannot be bought and sold. D. No, because Lumire and Ames have not previously associated in law practice, either as partners or otherwise.

3. C. Item I is correct under ABA Model Rule 72(a). Item II is correct. Anton's advertisement states that the "most" he will charge for "any type of legal work" is $100 per hour. If in fact he charges $125 for complicated legal work, his advertisement is false. [See ABA Model Rule 7.1] Item 111 is not correct; Anton has no obligation to disclose that other lawyers charge less than he does. If Anton's advertisement stated or implied that his fees are "the lowest in town," or something to that effect, that would be a different matter——but the facts stated in the question do not suggest any such statement or implication.

3. Attorney Anton advertised on the local television station. His advertisement stated in relevant part: "The most I will charge you for any type of legal work is $100 per hour, and if your problem is not complicated, the hourly fee will be even lower." Which of the following propositions are correct? I. Anton may advertise on the local television station so long as his advertisement is not false or misleading. II. If Anton charges $125 per hour for complicated legal work, he will be subject to discipline for using a misleading advertisement. III. If Anton's advertisement fails to state that some other lawyers in the community charge substantially lower fees, he will be subject to discipline. A Only II is correct. B. Only I and III are correct. C Only I and II are correct. D. Only II and III are correct.

3. B. The ABA Model Rules do not contain a blanket prohibition against lawyers dating their clients. However, ABA Model Rule 1.8(j) prohibits a lawyer from having a sexual relationship with a client unless' a consensual sexual relationship already existed between them at the time the lawyer-client relationship was commenced.

3. Charles Howard is a partner with a large international law firm. Charles finds one of his new clients, Tiffany Green, very attractive. Charles has been flirting with Tiffany and has asked her out to lunch and dinner on several occasions. A. Charles's personal life is his business; a lawyer's personal life is not regulated by the Model Rules. B. Charles has not yet violated the Model Rules, but his firm may prohibit its lawyers from dating clients. C. The Model Rules prohibit lawyers from dating clients and therefore Charles is subject to discipline. D. The Model Rules permit relationships between lawyers and clients so long as the relationship is consensual and no prejudice would result to the client.

3. A. Sanford should disclose the information to Rossi's counsel because Beaumont's conclusion suggests that the fire was not caused by a criminal act. [See ABA Model Rule 3.8(d).] Answers C and D are accordingly incorrect. Answer B is incorrect because a lawye—especially a prosecutor—should not ask a third party witness not to give relevant information to an adversary. [ABA Model Rule 3.4(f).]

3. Deputy District Attorney Sanford has been assigned to prosecute defendant Rossi for arson. Shortly after the fire was extinguished, a three- person team of arson experts was sent by the City fire department to determine the cause of the fire. The team concluded that the fire was set by a professional arsonist, and the team's report so states. Shortly before trial, Sanford learned that Beaumont, the youngest and least experienced member of the team, had originally concluded that the fire resulted from an explosion in the furnace. Beaumont had tried to convince the other two team members that his original conclusion was correct, but they ultimately prevailed, and Beaumont signed the report without dissent. Sanford does not plan to offer the report in evidence at trial, and he does not plan to call any of the three team members as witnesses. Rather, he plans to use the testimony of two independent experts to establish that arson caused the fire. Which of the following is the proper thing for Sanford to do concerning the information about Beaumont? A. Disclose it to Rossi's counsel, since it could be useful in Rossi's defense. B. Instruct Beaumont not to mention his original conclusion to anyone. C. Wait to see whether Rossi's counsel asks for the information in the regular course of criminal discovery. D. Do nothing about it since he does not plan to offer the report or the testimony of the team members at the trial.

3. A. The general rule is that a lawyer has no duty to serve just anyone who wants service and can pay the fee. While that general rule is subject to limitations, none of them apply here. There is no lack of skilled counsel in the community, the cause and client are not so unpopular as to shut off access to counsel, and Worthington has sufficient resources to obtain other counsel with more experience in the matter at hand. [See ABA Model Rules 6.1 and 6.2.]

3. Jason P. Worthington III is among the wealthiest men in New York society. When his son was arrested for selling illegal drugs to his prep school classmates, Worthington sought the legal services of the prestigious old firm of Bradbury & Crosswell. The Bradbury firm practices almost nothing but securities and banking law. For which of the following reasons may the Bradbury firm decline employment in the case? I. That Worthington is not among the firm's regular clients. II. That the firm is not experienced in criminal litigation. III. That Worthington can obtain better service at lower fees from lawyers with more experience in criminal litigation. IV. That the firm does not want to take time away from its regular work for a matter such as this one. A. Alloftheabove. B. None of the above. C. 111 only. D. II only.

3. D. All states require bar applicants to demonstrate good moral character. See generally Deborah Rhode, Moral Character as a Professional Credential, 94 Yale L.J. 491 (1985). A recent conviction for federal tax fraud is strong (though perhaps not conclusive) evidence that Samuel lacks good moral character. [See generally Hazard & Hodes §62.7.] As for item I, the Piper case, 470 US. at 274—288, suggests that State C could not refuse to admit Samuel simply because he plans to live. across the state line. As for item 11, membership in a radical political party is not, by itself, sufficient ground to deny admission to the bar. (Cf. SChware v. Board of Bar Examiners, 353 US. 232, 77 S.Ct. 752, 1 L.Ed.2d 796 (1957) (membership in the Communist Party).] As for item Ill, lack of US. citizenship is not, by itself, sufficient ground to deny admission to the bar. [See In re Griffiths, 413 US. 717, 93 S.Ct. 2851, 37 L.Ed.2d 910 (1973),]

3. Law graduate Samuel has passed State C's bar examination. For which of the following reasons could State C constitutionally refuse to admit Samuel to practice? I. Samuel plans to live in neighboring State D and to commute to work at a law office in State C. II. Samuel is an active member of the Founding Fathers Party, a small but vocal political organization that advocates radical realignment of the respective powers of the state and federal governments. III. Samuel is not a citizen of the United States. IV. Two years ago, Samuel was convicted of federal tax fraud. A. None of the above. B. Alloftheabove. C. II, III, and IV only. D. IV only.

3. C. Once it becomes apparent that Markler will not cooperate, Lattimer's best course of action is to take the matter up with Markler's corporate superior. the Executive Vice President. [See ABA Model Rule 1.13.] Answer C is preferable to answer D because C creates less risk of disrupting the corporate operations and revealing confidential information to outsiders. [See ABA Model Rule 1.13(b).] Answer B is incorrect because B would breach Lattimer's duty of confidentiality. Further. a competent lawyer would not need an advisory opinion from the Justice Department to know that phone calls between competitors about future prices raise antitrust problems. [See, e.g., United States 0. Container Corp. ofAmerica, 393 US. 333, 89 S.Ct. 510, 21 L.Ed.2d 526 (1969).] Answer A is incorrect because Lattimer's client is the corporation, not Markler. and she has a duty to warn her client of Markler's activity. [See ABA Model Rule 1.13(b).]

3. Lawyer Lattimer is on the in-house legal staff of Centennial Corporation, a major manufacturer of steel shipping containers. She regularly provides legal advice to Vice-President Markler, the executive in charge of sales and marketing. In the course of a routine preventive law project, Lattimer discovered that Markler had participated in a series of telephone conferences with his counterparts at the company's two main competitors. Further, she discovered that each such conference was promptly followed by an increase in the prices charged by the three companies. When Lattimer took this up with Markler, she first reminded him that she was not his personal lawyer, but rather the corporation's lawyer. Then she said: "If you have been discussing prices with our competitors, we may be in deep trouble. Your telephone conferences may violate the Sherman Antitrust Act, and that could mean civil and criminal liability, both for you and for the corporation. And, as you know, the corporation has a rule against rescuing executives who get in antitrust trouble." Markler responded as follows: "Ms. Lattimer, I know you're a good lawyer, but you don't know much about the real world. You can't run a business these days if you try to trample on your competition. Now don't worry yourself about my telephone conferences, because I'm sure you have better things to do with your time." If Markler remains uncooperative, which of the following expresses the proper course for Lattimer to take? A. Draft a careful, complete memorandum about the matter for her own files, and maintain her conversation with Markler in strict confidence. B. Describe the relevant facts in a carefully drafted letter to the Antitrust Division of the United States Department of Justice, request an advisory opinion on the legality of the described conduct C. Describe the entire matter to Markler's immediate corporate superior, the Executive Vice President, and advise him to put a step to Markler's telephone conferences. D. Describe the relevant facts in a memorandum to the corporate Board .of Directors and advise the Board that she will resign unless something is done to stop Markler

3. A. When two lawyers or law firms work on a case together, they frequently submit separate bills to the client, and nothing in the ABA Model Rules says this is improper. Answer B is not correct. ABA Model Rule 7.2(b) prohibits Alvarez from paying Leland for the referral. Answer C is not correct for the same reason that answer A is correct. The two lawyers could have worked out a suitable fee splitting arrangement here, but nothing requires them to do so. Answer D is not correct. The $1,000 referral fee makes the arrangement improper, even if it did not increase the total amount Holiday paid.

3. Lawyer Leland is admitted to practice only in Kentucky. He regularly represents Holiday Hotels, Inc., a Kentucky corporation with its principal offices in Lexington. Holiday was sued for trademark *infringement* in Oregon and Holiday asked Leland to oversee the case and to select anroiriate Oregon counse t0 (0 e ' '. Zp ' l . LblandL selected Oregon attorney Alvarez. and Alvaie aiddLheland $1,000 for the referral. Leland oversaw all of the work in the case, an d e conducted an of the discovery that took place in Kentucky- Alvarez con ucted all of the discovery that took place in Oregon. and he Prepared the case for tI'lal and served as trial counsel. At, the conclusion of the case. Leland and Alvarez submitted separate bills to Holiday for their respective services. Which of the following statements is correct? A. It was proper for Leland and Alvarez to blll Hollday separately, assuming that each bill was reasonable in amount. B. It was proper for Alvarez to pay Leland $110.00 for the referral. since the two lawyers shared the work and responSIblllty for the case. C. Leland and Alvarez are subject to discipline for failing to submit a single bill to Holiday. because. the two lawyers shared the work and responsibility for the case. D. The arrangement was proper, unless the total fee Holiday paid was higher than it, would have been absent the $1,000 referral fee.

3. B. Even in a contingent fee case, it is the client, not the attorney, who decides whether to accept or reject a settlement offer. Here, attorney A's duty is to inform R, S, and T of Us offer, even though A may think it is ridiculously low. [See ABA Model Rules 1.2 and 1.4; Hazard & Hodes §5.7, Illustration 5—5.] Furthermore, a lawyer who represents several clients on the same side of a case has additional duties when the adversary makes an aggregate settlement offer. The lawyer must fully disclose the entire offer to each of the several clients, and the clients must reach their own decision about accepting it or rejecting it, and about how to share it if they do accept it. [See ABA Model Rule 1.8(g).]

3. R, S, T, and U are four sellers of high-speed photo image reproductor disks. U falsely advertises its disks, and Us false statements injure R, S, and T by causing some of their customers to buy instead from U. But R, S, and T are not sure of the precise amount of business each lost. The three of them hire Attorney A to represent them in a suit against U for an injunction and damages, and A agrees to take the case for a 30% contingent fee. After extensive discovery, US attorney calls A with a settlement offer: U will consent to a court order enjoining U from using the allegedly false statements in future advertising, and U will pay a total of $100,000, in return for a full release of all claims by R, S, and T. In A's opinion, the consent order would adequately protect R, S, and T from future harm, but A believes in good faith that $100,000 is ridiculously low and would not compensate R, S, and T for their past losses. Which of the following is most nearly correct? A. A will be subject to discipline if he accepts U's settlement offer, because he does not believe in good faith that $100,000 would sufficiently compensate R, S, and T for their past losses. B. A will be subject to discipline if he does not let R, S, and T decide what to do about US settlement offer, even though he represents them on a contingent fee basis, and even though he believes that $100,000 is not enough to compensate them for their past losses. C. A may accept U's settlement offer, provided that he takes no more than 30% of it as his fee, and that he distributes the remainder equitably among R, S, and T. D. A may reject U's settlement offer, since he does not believe in good faith that $100,000 would sufficiently compensate R, S, and T for their past losses.

8. C. The question states that Landsman volunteered to represent CAH, not Cheng. When a lawyer advises one party to an arms-length transaction, the lawyer is not liable for negligence that injures the other party, except in the narrow situations described in Restatement of the Law Governing Lawyers § 51 (2000). [See Restatement § 51, comment c.] Answer A is not correct for the same reason. Answer B is not correct; working for free does not insulate a lawyer from malpractice liability. Answer D is not correct because the injury to Cheng would have been foreseen by a reasonably prudent lawyer.

3. The Community Association for the Homeless (CAH) is a non-profit charitable corporation that provides food and temporary shelter for homeless persons. CAH subsists on charitable donations and volunteer labor provided by members of the community. CAH owns a large old home in the downtown area, but it has virtually no other assets. Seeking to assist CAH in a time of financial need, Corliss Cheng decided to lend CAH $500,000, interest-free, for two years. Lawyer Landsman offered his services Without a fee to represent CAH in the transaction and to prepare the necessary loan papers. Cheng was not represented by a lawyer in the transaction. Landsman prepared a suitable promlssory note. The officers of CAH duly executed the note and presented it to Cheng in return for the $500,000. A year later, CAH was Overcome by financial disaster; the corporation was dissolved, and its creditors took over its few remaining assets. Cheng received only $2,000. Any reasonably competent general practitioner would have advised Cheng to Secure the interest-free loan by obtaining a deed of trust on CAH's large old home. Cheng sued Landsman for legal malpractice. Which of the following is most nearly correct? A. Landsman is subject to civil liability for malpractice in the suit brought by Cheng. B. Landsman is not subject to civil liability for malpractice because he did the legal work as a volunteer, not for a fee. C. Landsman is not subject to civil liability for malpractice because he did not purport to represent Cheng in the transaction. D. Landsman is not subject to civil liability for malpractice because the injury to Cheng was not foreseeable.

3. A. Carla's and Carl's disclosure to Anderson is confidential information, protected by ABA Model Rule 1.6. ABA Model Rule 3.3(a)(3) does not require disclosure for at least one and perhaps two reasons. First, since the adoption proceeding has come to a conclusion, the duty to disclose no longer applies. [ABA Model Rule 3.3(c).] Second, it is not clear from the facts stated in the question that Anderson "offered false evidence" in the adoption proceeding. She did offer the marriage certificate, but that document was not false, and Anderson had no reason at the time to believe that Carla and Carl were not validly married. Answers C and D are incorrect for the same reasons. Answer B is incorrect as well. "Continuing fraud" is a foggy concept at best, but even assuming that Carl and Carla are committing a continuing fraud by keeping their adopted child and living together as husband and wife, there is no reason to assume that Anderson's advice would "assist" them in continuing the fraud.

3. The law of State X requires child adoptions to be approved by the court. Further, it prohibits cohabiting couples from adopting a child unless they are validly married. Attorney Anderson represented clients Carla and Carl in an adoption proceeding. They assured her that they were validly married. Among the papers she presented to the court in connection with the adoption proceeding was copy of Carla's and Carl's Certificate of Marriage, duly certified by the custodian of public records. In due course, the court approved the adoption. A year later, Carla and Carl returned to Anderson's office. Carla explained to her as follows: "When we came to you about the adoption, there's something we didn't tell you, because we didn't want to get into lots of complications. Carl was married once before. His wife moved out, and he hasn't heard from her since. When he and I began dating, we fell in love so fast that there wasn't time for him to go through a divorce before we got married. We don't want to do anything that might risk losing our child, but this has been bothering us, and we thought we should come to you for advice." What is the proper course of action for Anderson to take? A. Advise Carla and Carl about the legal effect of the prior marriage on their current status and on the adoption. B. Decline to advise Carla and Carl, thus avoiding the assistance of a continuing fraud. C. Advise Carla and Carl to reveal their fraud to the court that approved the adoption, and warn them that she will do so if they do not. D. Bring the matter to the attention of the court that approved the adoption, and let the court decide what remedial action is appropriate in the circumstances.

4. D. The comments to ABA Model Rule 8.4. as well as ABA Model Rule 3.4(e) and 4.4(a), all apply here. Answers A. B. and C are therefore incorrect.

4. Aaron Campbell is representing Bradley Whitehouse in a civil lawsuit in which Whitehouse allegedly wrongfully terminated the employment of Brenda Gale, who had been Whitehouse's housekeeper. Gale alleges she was fired after rebuffing Whitehouse's sexual advances. Whitehouse is a prominent, wealthy businessman. At trial, Campbell questioned Gale repeatedly about her sexual orientation, suggesting that Gale was a lesbian who filed the lawsuit as an opportunistic attempt to extort damages from Whitehouse. Campbell's line of questioning stemmed from his Observation that Gale was 31, never married, and unattractive. A. Campbell was required to provide a defense for his client, so he cannot be subject to discipline. B. Although offensive, nothing in the ABA Model Rules prohibits such tactics. C. Only the judge presiding over the trial has any obligation to address bias occurring in the courtroom. D. Campbell is subject to discipline

4. B. Under ABA Model Rule 1.8(e), Aragon is subject to discipline. The $7,500 was for medical expenses, not litigation expenses, so answer B is correct. Answer A is not correct. This is the kind of case that lawyers commonly take on a contingent fee basis. Answer C is not correct. As you will learn in a later chapter, it is proper to pay a reasonable fee to an expert witness. [See Comment to ABA Model Rule 3.4.] Answer D is not

4. After Carlson was injured in a car wreck, he was treated in the hospital for twelve days by physician Patino: she billed him $7,500 for her medical services. The wreck put Carlson out of work, and he had no way to pay Patino's bill. He hired attorney Aragon to sue the person who caused the wreck: in a written fee agreement, Aragon promised to do the work for a contingent fee. Aragon decided that Patino would make a good expert witness in the case. Aragon and Carlson agreed that Aragon would lend Carlson $7.500 to pay Patino's medical bill and that Aragon would advance the money needed to pay Patino at $100 per hour for the time she spent preparing to testify and testifying as an expert witness. Carlson agreed to pay back Aragon at the conclusion of the case. Which one of the following statements is correct? A. Aragon is subject to discipline for taking the case on a contingent fee. B. Aragon is subject to discipline for agreeing to lend Carlson the $7,500. C. Aragon is subject to discipline for participating in an agreement to pay a witness for giving testimony. D. Aragon is subject to discipline for agreeing to advance the money needed to pay Patino's expert witness fee.

4. A. When a lawyer represents one party to litigation, the lawyer is not liable for negligence that causes injury to the adversary party. [See Restatement, supra, § 51, comment c.] Further, nothing in the question suggests that Applegate was negligent in the first place. A lawyer is liable to the adversary for intentional misconduct, for example, abuse of process. If Delta had sued Applegate for abuse of process, it would have had to prove that Applegate intentionally pursued a claim that he knew was baseless. The question states that Applegate believed that Delta had in fact unlawfully discriminated against Cortez, even though Applegate was pessimistic about Cortez's chances of winning at trial. Answers B and D are not correct for the same reason. Answer C is not correct. If Applegate had owed a duty of care to Delta, and if Applegate had acted negligently, the actual cause element would have been easy for Delta to satisfy.

4. Attorney Applegate represented client Cortez as plaintiff in an employment discrimination action against Delta Corporation. After considerable pretrial discovery, Applegate and Cortez concluded that Delta had indeed unlawfully discriminated against Cortez but that they probably would be unable to convince a jury of that fact. They decided not to pour any more money into pretrial discovery and to trust to good luck when the case came to trial. Before the case was set for trial, Delta moved for summary judgment. Delta's motion was granted, and the case was dismissed. Then Delta sued Applegate for legal malpractice, alleging that he was negligent in advising Cortez to maintain the suit against Delta and that Delta had been injured to the extent of its litigation costs and attorney fees. In Delta's action against Applegate, which of the following is most nearly correct? A. Applegate is not subject to civil liability for malpractice, even if he lacked a good faith belief that Cortez would win at trial. B. Applegate is subject to civil liability for malpractice if he lacked a good faith belief that Cortez would win at trial. C. Applegate is not subject to civil liability for malpractice because his conduct was not the actual cause of Delta's injury. D. Applegate is subject to civil liability for malpractice if he was negligent in advising Cortez to oppose Delta's motion for summary judgment.

4. D. Answer D is correct under ABA Model Rule 1.6(b)(2). Answers A and C are not correct. The crime of extortion (blackmail) includes the obtaining of money by inducing fear in the victim. One common way of inducmg that fear is by a threat to reveal the victim's secret, and Aquino's letter seems well designed to do that and is therefore improper. [See ABA Model Rule 8.4(b)—(d).] Answer B is not correct. Comment 5 to ABA Model Rule 1.5 encourages lawyers to consider resolving a fee dispute by arbitration or mediation where it is available, but (unless state law makes arbitration mandatory) a lawyer would not be subject to discipline for bringing suit to collect a fee.

4. Attorney Aquino defended Dempsey in a criminal assault case. Before trial, Dempsey told Aquino in confidence that he beat up the victim without provocation. Due to Aquino's hard work, coupled with a stroke of luck, the jury found Dempsey not guilty. Then Dempsey refused to pay Aquino's fee. Aquino wrote to Dempsey as follows; "The jury found you not guilty, but your victim can still sue you for civil damages. If you do not pay my fee, and if I have to sue you to collect it, I will have to reveal the whole truth in open court, to explain why the amount of my fee is reasonable. Think this over carefully. I hope to receive your check by return mail." Which of the following is most nearly correct? A. Even though heavy-handed, Aquino's letter was proper because he was simply explaining to Dempsey the consequences of refusing to pay the fee. B. If Aquino sues Dempsey to collect the fee, Aquino will be subject to discipline because a lawyer is prohibited from using a civil suit to collect a fee. C. Aquino's letter was proper because a lawyer is required to settle fee disputes amicably if possible. D. If Aquino sues Dempsey to collect the fee, Aquino may reveal Dempsey's confidential communications, but only to the extent necessary to establish his claim against Dempsey.

4. A. If Daniels sent the carbon copy to Parker, he would violate ABA Model Rule 4.2, even if he is positive that Paxton did not convey the settlement offer to Parker. If Paxton has failed to convey the offer to Parker, then Paxton has violated his duty to keep Parker informed and to let Parker make the important decisions in the case. [See ABA Model Rule 1.2(a).] But the proper remedy for that violation is for Daniels to bring Paxton's conduct to the attention of the judge to whom the case is assigned. not to deal directly with Parker. [See ABA Informal Op. 1348 (1975) (copy of settlement offer sent to represented adversary in civil case); see also ABA Informal Op. 1373 (1976) (copy of offer to plea bargain sent to represented criminal defendant).] Answers B, C, and D are incorrect for the same reasons.

4. Attorney Paxton represents plaintiff Parker on a contingent fee basis in an action against Dougal Corp. for breach of an alleged employment contract between Dougal Corp. and Parker. Attorney Daniels represents Dougal Corp. in the matter. Dougal Corp. instructs Daniels to offer the plaintiff $35,000 to settle the case, and Daniels duly telephones Paxton and makes the offer of settlement. Paxton says he will take it up with Parker and get back to Daniels in due course. When Daniels hears nothing for two weeks, he calls Paxton back, but Paxton refuses to take Daniels' telephone call. Then Daniels writes Paxton a formal letter, re-making the settlement offer, requesting that Paxton consult Parker about it, and requesting a prompt response. Again, Daniels hears nothing from Paxton. Finally Daniels develops a strong suspicion that Paxton has not communicated the settlement offer to Parker. May Daniels send Parker a copy of his letter to Paxton? A. No, because Daniels is not allowed to communicate directly with Parker. B. Yes, if Daniels reasonably believes that Paxton has failed to communicate the settlement offer to Parker. C. No, since Daniels cannot be sure that Paxton failed to communicate the settlement offer to Parker. D. Yes, because the copy would simply advise Parker of Daniels' prior communication with Paxton.

4. D. The financial interest held by Prentice is imputed to all lawyers in the firm. [See ABA Model Rule 1.10(a).] Since that interest is in conflict with the Alliance's interests, Tillis cannot accept the case without first disclosing the conflict to the Alliance and obtaining the Alliance's consent. [See ABA Model Rule 1.7(b).] Item II is not correct. Vista del Oro is not now, and has never been, a client of the firm. Thus Vista del Oro's sentiments about the conflict are irrelevant. Neither Item III nor Item IV is a necessary condition of allowing Tillis to represent Alliance. If the Alliance consents after full disclosure of the conflict, that is sufficient; Prentice need not rid himself of his interest in Vista del Oro. Note that an alternative way to resolve the conflict of interest might be for Prentice to resign as a director and to sell his seven home sites, but that is not among the four answers offered in this question. Further, even if Prentice were to get rid of his interest in Vista del Oro, many firms would still disclose the situation to the Alliance, simply as a matter of good client relations.

4. Attorney Tillis is a partner in the 138 person firm of Dahlberg & Sneed. The Citizens' Alliance for Coastal Preservation has asked 'l'illis to represent the Alliance in a public interest law suit against Vista del Oro, Inc., a real estate developer. Vista del Oro owns several thousand acres of beautiful coastline, about an hour's drive from the largest city in the state. It is building vacation homes to sell to the public. When the project is complete, the entire area will be fenced off to prevent access by non-owners. The Al,liance seeks to force Vista del Oro to provide access paths across the property, so that members of the public can get from the state highway to the public beaches. Attorney Prentice is also a partner in Dahlberg & Sneed. He is a member of the Board of Directors of Vista del Oro, and he owns seven of the vacation home sites as a personal investment. No Dahlberg & Sneed lawyer has ever represented Vista del Oro, and none will do so in the present case. After careful consideration, Tillis has concluded that his representation of the Alliance would not be adversely affected by Prentice's interest. Which of the following conditions must be met if Tillis is to avoid being subject to discipline for representing the Alliance? I. The Alliance consents after full disclosure. II. Vista del Oro consents after full disclosure. III. Prentice resigns as a director of Vista del Oro. IV. Prentice sells his seven home sites. A. Alloftheabove. B. III only. C. I and II only. D. I only.

4. D. ABA Model Rule 3.4(b) prohibits a lawyer from counseling or assisting a client (or any other witness) to testify falsely. [See also 18 U.S.C. §1622 (1982) (subornation of perjury); Tedesco v. Mishkin, 629 F.Supp. 1474 (S.D.N.Y.1986).] Answer B is not correct. Lawyers should, and commonly do, talk with clients and other witnesses about the testimony they will give. But. as one judge put it long ago, the lawyer's task is "to extract the facts from the witness, not to put them into him." [In re Eldridge, 82 NY. 161 (1880).] Answer A is not correct. A lawyer should represent a client with zeal, but only within the bounds of the law, which includes the rules of legal ethics. The assertion in Answer C is accurate, but it is misapplied here. If the lawyer's objective is to bend the witness's testimony, ABA Model Rule 3.4(b) applies, even if the bending takes the form of a lecture on the law. [See Wolfram, § 12.4.3, at 648.]

4. Client Curtis hired lawyer Lomax to defend him in a civil antitrust action brought by Pucci, a former retail distributor of products that Curtis sold to Pucci. Pucci alleges that Curtis terminated him as a distributor because Pucci sold the products below a minimum retail price set by Curtis. Pucci further alleges that the termination resulted from a secret agreement between Curtis and other distributors that Curtis would terminate any distributor who sold below the minimum retail price. Assume that such an agreement, even if coerced, would violate the antitrust law. When Lomax was preparing Curtis for his deposition, Lomax asked Curtis why he terminated Pucci. Curtis answered: "Because Pucci was a price cutter. My other distributors pressured me to do it." At that point, Lomax said: "If you say that at your deposition, you will lose this case. Before you say more, let me tell you about the law that applies here. If you, using your own business judgment, terminated Pucci because he was not doing a good job, or because he was not displaying or advertising your products effectively, then the termination would be lawful. If, on the other hand, you let your other distributors talk you into terminating him because his prices were too low, the termination would probably be unlawful. Now go back to your office, refresh your memory of this event, and we will talk again tomorrow morning about your reasons for terminating Pucci." Was Lomax's conduct proper? A. Yes, because a lawyer has a duty to represent a client with zeal, using all lawful means to achieve the client's objectives. B. No, because it is improper for a lawyer to discuss the substance of a client's testimony with the client before the client testifies. C. Yes, because a lawyer has a duty to advise a client fully about the law that applies to a matter. D. No, because it is improper for a lawyer to invite a client to give false testimony.

4. A. Assuming that Items 1, II, and IV are truthful misleading, they are proper under ABA Model Rules 7.1 and 7.2. Item III is proper under ABA Model Rule 7.4(d); the bar of this state does not approve certifying agencies, but the ABA has accredited the certifying organization.

4. Lawyer Del Campos practices in a town in which 25% of the people are Mexican-American and another 20% have recently immigrated to the United States from Mexico. The bar of his state does not certify specialists in any field of law nor does it approve pI'iVate 0rg3n1._l'.at1'0n8 that cert'if'y legal specialists. However, Del Campos has been certified aS. a SpeCIaIISt in immigration law by the American Association of Immlgrc'atlon Attorneys, ,3 private organization accredited by the ABA. Del Campos wants to put an advertisement in the classified section of the local telephone book. Which of the following items of information may he include in his advertisement? I. That he "serves clients who are members of the Continental Prepaid Legal Service Plan." II. That he "speaks Spanish." III. That he has been "certified as an immigration specialist by the American Association of Immigration Attorneys." IV. That he can "arrange credit for fee payments." A. All of the above. B. None of the above. C. I, II, and IV only. D. I and III only.

4. C. A lawyer is subject to discipline for engaging in conduct involving dishonesty, fraud, deceit, or misrepresentation. [ABA Model Rule 8.4(b) and (c); see also Cal.Bus. & Prof.Code § 6106.] Leon's conduct was dishonest for two reasons: first, he intentionally cheated the phone company, and second, he lied to the judge. On the other hand, Leona's unwitting violation of the mushroom statute does not strike us as the kind of conduct that shows unfitness to practice law, or untrustworthiness, or dishonesty.

4. One of lawyer Leon's clients gave him a "Little Yellow Box," an electronic device that enables one to make free long distance telephone calls from a pay phone. Leon used it occasionally to call his mother in Des Moines. Use of such a device is a misdemeanor under the applicable state law. Leon was arrested for using the device. At his trial, he denied ever having it in his possession. The judge did not believe him, found him guilty, and fined him $1,000. That same week, Leon's law partner, Leona, went backpacking in the mountains. She was arrested by a Forest Ranger for violating a state statute that makes it a misdemeanor to pick mushrooms in a state forest during certain months. Leona did not know about the statute. Leona pleaded guilty to the charge, and the judge fined her $1,000. A. Both Leon and Leona are subject to discipline. B. Neither Leon nor Leona is subject to discipline. C. Leon is subject to discipline, but Leona is not. D. Leona is subject to discipline, but Leon is not.

4. C. One of the limitations on an attorney's ordinary freedom to turn down cases is where the court asks the attorney to serve as appointed counsel. [ABA Model Rule 6.2] An attorney should not seek to be excused from taking a court appointed case, except for a compelling reason. Items 1, III, and IV do not present compelling reasons. But Item II, concerning the risk of unreasonable financial harm, does present a compelling reason, according to ABA Model Rule 6.2(b).

4. When attorney Hodges graduated from law school three years ago, she opened a solo practice in a small rural community close to the state's major prison. Her primary interests are family law and real estate law. Her practice is growing very slowly, despite her long work hours. She is barely able to make financial ends meet. The presiding judge of the local State District Court has asked her to serve as court-appointed counsel in a civil action that was originally filed in propria persona by an indigent inmate of the prison. From the roughly drawn complaint, the presiding judge believes there may be some merit in the inmate's allegations of brutality by some of the guards and gross neglect on the part of the warden. State law allows attorney fees to be awarded to a plaintiff in a civil action of this type, but only if the plaintiff is victorious. Attorney Hodges realizes that she will not be paid for her work if she loses the case, and she is very concerned about the financial loss she may suffer if she takes time away from her regular practice. Further, she is worried about harming her reputation because the warden and many prison employees form the nucleus of her community. Which of the following statements are correct? I. She may decline to serve on the ground that her practice primarily in the fields of real estate and family law. II. She may decline to serve if she believes in good faith that she cannot reasonably take the financial risk involved. III. She may decline to serve if she believes in good faith that to serve would seriously injure her reputation in the community. IV. She may decline to serve if she believes in good faith that some of her present clients will be offended if she takes the case. A. All of the above. B. None of the above. C. II only. D. I, II, and IV only.

4. D. Although a generous reading of Rule 1.7 might suggest that Anna may represent both Wendy and Harry if she reasonably believes that she can serve both effectively, and if both of them consent to the joint representation after Anna explains to them the disadvantages of being represented by only one lawyer, the likelihood of future conflict is high. [See Hazard & Hodes § 11.4, Illustration 11—1 (noting that "the division of their marital property is inherently a zero-sum prOposition. Even in today's no-fault divorce regimes, the spouses will almost certainly be required to appear as nominally adverse parties in an actual court proceeding and to submit their agreement for formal court approval") See generally Debra Lyn Bassett, Three's A Crowd: A Proposal to Abolish Joint Representation, 32 Rutgers L.J. 387, 425—427 (2001); Nancy J. Moore, Conflicts of Interest in the Simultaneous Representation of Multiple Clients, 61 Texas L.Rev. 211, 245—58 (1982).] Answers A and C are incorrect for the same reason that answer D is correct. Answer B is incorrect because it does not meet the requirements of the rules cited above; further, obtaining a client's advance promise not to sue for legal malpractice is itself a disciplinary Violation. [ABA Model Rule 1.8(h).]

4. Wife Wendy and husband Harry ask attorney Anna to represent both of them in a dissolution of marriage proceeding in a state that allows "no-fault" dissolution. The couple has no children. Wendy is a successful young pediatrician, and Harry is an unemployed computer programmer. They want Anna to represent both of them because separate lawyers may cost more and may stir up antagonism, and because they hope Anna can help them reach a property and support agreement that is mutually acceptable. Which of the following is most nearly correct? A. If Anna represents both, she will be subject to discipline for counseling both sides in an adversary proceeding. B. Anna may represent both, provided that she obtains from them separate covenants not to sue her later for legal malpractice. C. If Anna represents both, she will be subject to discipline because Wendy's and Harry's interests are in present, actual conflict. D. Anna should not represent both, because it is unlikely that she can serve both effectively.

5. A. Item B is incorrect; ABA Model Rule 1.5(a) states that one factor a lawyer may consider in setting a fee is "the amount involved and the results obtained." Item C is incorrect; a portion of Draxco's check did belong to Arnstein, but Arnstein's portion had not yet been determined. In that situation, it was proper to put the entire amount in the client trust account. [See ABA Model Rule 1.15(c); RES'I'ATEMENT (THIRD) OF THE LAW GOVERNING LAWYERS §44, comment f (2000).] Item D is incorrect; where there is a fee dispute, it is proper to keep the disputed funds in the client trust account until the dispute is settled. [1d,]

5. Attorney Arnstein agreed to represent client Clemens in 3 products liability suit against Draxco, Inc. Clemens refused to discuss Arnstein's fee at the outset of the case; rather, Clemens insisted on a provision in the retainer agreement that Arnstein would do the work "for a reasonable. fee, to be deducted from the proceeds" of the case. After a long period of discovery. Arnstein arranged a very favorable settlement between (./'leniens and Draxco. Draxco paid the $175000 settlement by a check made payable to Arnstein. Arnstein immediately deposited the check in his client trust account and invited Clemens to come by the office to settle their affairs. When Glenn-11s arrived, Arnstein gave him a bill for $25,000. He computed that amount by multiplying the number of hours he spent on the case (.'hf'0) times his normal hourly rate ($65). and adding an extra $2,250 because of the generousness of the settlement he had achieved for Clemens. When Clemens looked at the bill, he turned scarlet and began to shout that the fee was. outrageously high. Arnstein explained the basis of his charge. and he offered to arbitrate the matter through the local bar association, but Clemens refused. When Clemens demanded immediate payment of the entire $175,000, Arnstein gave him a check, drawn on his client trust account, in the amount of $150,000. Arnstein kept the other $25,000 in his client trust account, pending ultimate resolution of the fee dispute. Which one of the following statements is correct? A. Arnstein's handling of the matter was proper. B. Arnstein is subject to discipline for charging Clemens more than his normal hourly rate. C. Arnstein is subject to discipline for depositing the entire proceeds of Draxco's check in his client trust account. D. Arnstein is subject to discipline for keeping the $25,000 in his client trust account pending resolution of the fee dispute.

5. B. ABA Formal Opinion 87—353 (1987) and its predecessor, ABA Formal Opinion 287 (1953), use a version of this hypothetical to illustrate the hair-fine balance between zealously representing a client and actively misleading a court. Bear in mind that this hypothetical involves the criminal process—the prosecutor carries the burden of proof, and the client is entitled to remain silent and to have the effective assistance of counsel. In this hypothetical, the adversary process has failed to produce the truth, and the client will get a windfall gain. If the lawyer responds as in Item I or II, the lawyer has not actively misled the court. True, the lawyer has failed to correct the court's mistaken belief, but that is deceit only if there is a duty to speak out, and there is no duty here. In Item 111, the lawyer asserts that his client's "record is clean." That is the literal truth: the record is clean, though the client is not. Nonetheless, it seems to us that in Item III the lawyer has stepped over the line and has actively misled the court in violation of ABA Model Rule 3.3(a)(1). In Item IV, the lawyer has corrected the court's mistake, but he has breached the duty of confidentiality and is subject to discipline under ABA Model Rule 1.6. A similar hypothetical is discussed in Hazard & Hodes § 29.9—29.10; see also Joan C. Rogers, Candor Toward Tribunals, 25 ABA/BNA Law. Man. Prof. Conduct 174, 175—76 (2009).

5. Attorney Arossio was defending Doyle in a drunk driving case. The state's drunk driving statute specifies a fine up to $1,000 for a first offense. For a second offense, it specifies a fine up to $10,000, plus a mandatory jail sentence of 60 days up to one year. Doyle told Arossio in confidence that he had one prior conviction for drunk driving. Arossio consulted the public records and found that Doyle's prior conviction had never been properly recorded. Doyle decided to plead guilty. The hearing transcript shows the following colloquy: The Court: Your guilty plea will be accepted, Mr. Doyle. Ms. Prosecutor, are there priors in this case? Prosecutor: No, your Honor. The People ask the maximum fine of $1,000. The Court: Very well. Mr. Arossio, since your client is a first-timer, I'm inclined toward a fine of $750. Is that acceptable? For which of the following responses would Arossio be subject to discipline? I. Yes, thank you, your Honor. II. My client will accept the court's judgment, your Honor. III. Since my client's blood-alcohol level wasn't much above the mark, and since his record is clean, I would ask your Honor for a fine of not more than $500 IV. There's been a mistake, your Honor. My client has a prior conviction that does not appear on the record. A. None of the above. B. III and IV only. C. I, II, and III only. D. IV only.

5. D. Answers A, B and C are incorrect under the principles expressed in the Zauderer and Shapero cases. Question 5 also raises a different ethics issue that you will study later in this book. You will learn that ordinarily a lawyer should not serve as trial counsel in a case where he or she is "likely to be a necessary witness." Salmon saw the accident and is therefore a potential witness, but he is probably not a "necessary" witness because he was only one of a crowd of people that saw the accident.

5. Attorney Salmon published a brochure entitled, "What to Do When You Are Injured." It contains accurate, helpful information on obtaining medical treatment, recording details of the event, notifying insurance companies, not making harmful statements, and the like. The cover of the brochure identifies Salmon as a "Personal Injury Attorney" office address and telephone number. One afternoon, Salmon was standing in a crowd of people that saw a pregnant woman knocked down in a pedestrian crosswalk by a speeding car. A few days later, Salmon mailed the woman a copy of his brochure, together with a letter stating that he had witnessed the accident and was willing to represent her for a reasonable fee should she wish to sue the car driver. The outside envelope stated that the envelope contained "Advertising Material." The bar in Salmon's state does not have a 30-day waiting period of the kind involved in Went For It, Inc., above. A. Salmon is subject to discipline, both for sending the woman the brochure and for sending her the letter. B. Salmon is subject to discipline for sending the woman the letter, but not for sending her the brochure. C. Salmon is subject to discipline for offering his legal services, for a fee, to a person who was not a relative, client, or former client. D. Salmon's conduct was proper.

5. A. ABA Model Rule 1.6(a) makes a lawyer subject to discipline for revealing a client's confidential information. The future crime exception does not apply here. Colbert has revealed past crimes, not an intent to commit future crimes. She has now moved out of the house and is living in a distant town, and the facts stated in the question do not suggest that she intends to abuse the children further. Answer B is incorrect; Lamb has no discretion here. Answer C is incorrect. The state statute is directed to physicians and psychotherapists, not to attorneys. Indianapolis Bar Ass'n Op. 1—1986 (1986) involves a similar statute and holds that an attorney has no duty to report past instances of child abuse. Answer D is incorrect because the crimes are past crimes, not future crimes.

5. Client Colbert has retained lawyer Lamb to represent her in divorce proceedings instituted by Colbert's husband. Colbert has moved out of the family home and is living in a distant town; she no longer sees her husband or their children. Colbert tells Lamb in confidence that, before the separation, she had been physically abusing the children. A state statute requires physicians and psychotherapists to report to the police all suspected cases of child abuse. The statute makes no mention of attorneys. Which of the following is most nearly correct? A. If Lamb reports the child abuse to the police, he will be subject to discipline. B. Lamb may report the child abuse to the police if he believes that the interests of justice will be served by doing so. C. Lamb must report the child abuse to the police, because the state policy favors the protection of children. D. Lamb must report the child abuse to the police, because child abuse is a crime that may result in death or serious bodily injury.

5. D. Limpett's direct contact with Victoria violates ABA Model Rule 4.3. A lawyer may talk with a potential adversary who is not represented by counsel, but in doing so the lawyer must not purport to give the person legal advice (other than to retain counsel if that is appropriate). [See ABA Model Rule 4.3.] Note that Limpett's advice to Victoria is also misleading in part. If Crebs were held liable to Victoria, the so-called "collateral source" rule would probably allow her to collect her medical expenses from him, even though those expenses were covered by her own health insurance. Answers A, B, and C are incorrect for the reasons explained above.

5. Crebs had an automobile accident in his sports car, injuring his girlfriend. Victoria, who was riding in the front seat without her seatbelt fastened. Crebs consulted lawyer Limpett about his possible legal liability to Victoria. After making sure that Victoria had not already retained counsel, Limpett went to visit her, to find out how badly she was injured and to obtain her description of what happened the night of the accident. Victoria asked Limpett whether he thought she should make a claim against Crebs. Limpett gave her his honest opinion: litigation can be costly and time-consuming, and Crebs' liability was debatable. Further, he told her, since her medical expenses were fully covered by her own health insurance, she had little to gain by suing Crebs. A. Limpett's conduct was proper, since Victoria was not represented by counsel when Limpett spoke with her. B. Limpett's conduct was proper, because he gave Victoria his honest opinion about the matter. C. Limpett's conduct was proper, provided that his Visit with Victoria was an overture to a good faith settlement of the matter. D. Limpett is subject to discipline, even if his ultimate objective was to reach a fair settlement of the matter.

5. D. Item I is correct. [See ABA Model Rule 1.1.] It would be 'E proper for Abrams to charge Carmondy a nominal fee for finding a = suitable specialist; to find a suitable expert takes time and requires careful judgment. As a practical matter, however, many lawyers in Abrams" position would elect not to charge a regular client for this service. Item II is correct. [See Comment 2 to ABA Model Rule 1.1.] Item III is not correct because it fails to mention the need to obtain Carmondy's written consent about the arrangement. [See ABA Model Rule 1.5(e).] Item IV is correct. [See Comment 2 to ABA Model Rule 1.1, which speaks ofstudy to become competent to handle a case.]

5. For many years attorney Abrams has done all of the routine business law work for Carmondy Corporation. Now Carmondy has asked him to represent it in negotiating a contract to supply electronic components to the US. Navy. Abrams knows nothing about government contract law except that it is a highly specialized field governed by a mass of technical regulations. Which of the following would be proper for Abrams to do? I. To decline to represent Carmondy, and to charge Carmondy a nominal fee for finding Carmondy a lawyer who specializes 1n government contract law. II. To agree to represent Carmondy, provided that Carmondy W111 consent to the association of a lawyer who specializes in government contract law. III. To agree to represent Carmondy, and then to subcontract the substantive legal work to a lawyer who specializes in government contract law. IV. To agree to represent Carmondy, intending to master the field of government contract law with reasonable speed and efficiency. A. All of the above. B. None of the above. C. II or III only. D. I, II, or IV only.

5. B. Under the ABA Model Rules, the starting point of the analysis is Rule 1.7. Rule 1.7 would not bar Smithers from serving as defendant's trial counsel, provided that the defendant consents after full disclosure of the drawbacks—Smithers' effectiveness may be somewhat reduced by his dual role as trial counsel and witness. ABA Model Rule 3.7 would not bar Smithers from serving as defendant's trial counsel, provided that the authenticity of the FTC letter is uncontested. [Hazard & Hodes § 33.7.] Finally, Hillner could serve as trial counsel even if Smithers could not, because this kind of conflict is not imputed to other lawyers in the firm. [ABA Model Rule 3.7(b).]

5. In a private treble damage case arising under the federal price discrimination law (the Robinson-Patman Act), the defendant wants to prove that it had a good faith belief that its pricing system was lawful. As evidence of its good faith, the defendant wants to prove that, five years ago, the Federal Trade Commission carefully reviewed the defendant's pricing system and decided not to institute proceedings against the defendant. The proof of this is a letter from the FTC to defendant's lawyer, Smithers. At the treble damage trial, the defendant will need Smithers' testimony to authenticate the letter-«Smithers will simply testify that he received the letter from the PFC. The defendant wants Smithers and his law partner, Hillner, to serve as its trial counsel in the treble damage case. Which of the following is correct? A. Neither Smithers nor Hillner may serve. B. Both Smithers and Hillner may serve. C. Only Hillner may serve. D. Smithers and Hillner may serve, but only if the plaintiff consents.

5. B. ABA Model Rule 1.16(b)(4) permits a lawyer to withdraw when the client "insists upon taking action that the lawyer considers repugnant or with which the lawyer has a fundamental disagreement." Here the officers of the union are asking Yeager to include a provision with which she strongly disagrees and considers inconsistent with the best interests of the local's members. Therefore, she is entitled to withdraw, assuming that she takes the ordinary steps to protect her clients interest upon withdrawal. [See ABA Model Rule 1.16(d).]

5. Lawyer Yeager has been retained by the officers of Amalgamated Finishers and Patternworkers Union, Local 453, to draft a new set of bylaws for the local. Yeager strongly disagrees with one of the provisions the officers want included in the new bylaws. The provision would deny members of the local the right to vote on some issues that involve the expenditure of union funds. Although Yeager believes that the provision is lawful and consistent with the national union charter, she believes it would be unwise and inconsistent with the best interests of the members of the local. If the union can obtain other counsel without serious loss, may Yeager withdraw from the matter. A. Yes, but only if she obtains the consent of her client. B. Yes, because her client is asking her to do something that is against her best judgment. C. No, because she is obliged to carry out the lawful objectives of her client. D. No, unless her client has breached the agreement under which she agreed to perform the work.

5. A. The relevant provisions are ABA Model Rules 1.9 and 1.10. The service station dealers' antitrust case is "substantially related" to the proposed antitrust legislation. Further, DePew has received confidential information from Transpac concerning the relationships between the integrated oil companies and their retail service station dealers. That information is clearly relevant to the dealers' antitrust case. Thus, DePew himself would be barred from taking the case, and the ordinary rule would also bar all of the other lawyers in DePew's firm. [See ABA Model Rule 1.10(a).] DePew's firm might try to invoke the so-called "ethical wall" or "screening" theory, promising that DePew would be screened off from the service station dealers' case and would share no part of the fees earned in that case. [Cf. Haagen-Dazs v. Perche Nol, supra] However, the screening provision of ABA Model Rule 1.10 applies to situations "aris[ing] out of the disqualified lawyer's association with a prior firm"— and here there is no prior firm.

5. Tillingham, Wadsworth & DePeW is a sprawling corporation law firm with 200 partners, 600 associates, and branch offices in eight major cities. Reynard DePew is the senior partner in charge of the firm's Washington, D.C., branch office. A year ago, he was retained by Transpac Oil Company to prepare some Transpac executives to testify before a Senate committee in opposition to proposed antitrust legislation that would require all integrated oil companies to divest themselves of their retail service stations. In connection with this work, DePew received truckloads of confidential documents from Transpac concerning competitive conditions in the retail end of the oil industry. DePew did not share this confidential information with anyone in the firm's Denver branch office, nor did he ever discuss the matter with anyone in the Denver office; indeed, no one in the Denver office even knew that DePew was working on the matter. Eight months after the matter was concluded, the Independent Service Station Dealers of America asked the firm's Denver office to represent it as plaintiff in an antitrust action against nine major integrated oil companies, including Transpac. May the Denver office accept the case without Transpac's consent? A. No, because the case is substantially related to the work DePew did for Transpac. B. Yes, because the case is not substantially related to the work DePew did for Transpac. C. Yes, provided that the firm concludes that it can effectively represent the Dealers Association and that DePew is screened off from the case and does not share any fees earned in the case. D. Yes, because the Denver office never received any of Transpac's confidential information from DePew.

5. A. ABA Model Rule 8.1 requires candor of a bar applicant on an application questionnaire. [See also CRPC 1—200] Answer B is not correct; a bar applicant can overcome a prior criminal conviction by demonstrating rehabilitation. Answer C is not correct; the prior conviction is relevant to Sabrina's present moral character, although she can be admitted to practice if she demonstrates rehabilitation. Answer D is not correct; no case has extended the constitutional right of privacy this far, although it has been argued that privacy values ought to be given more attention in bar admission matters. [See Rhode, supra, at 574—84; John Gibeaut, Perils of Prozac Probes,' 86 A.B.A.J., Feb. 2000, at 20 (Americans with Disabilities Act may limit bar admission questions on mental health, past addiction, and the like).]

5. When law student Sabrina was 17-years—old, a juvenile court in State A convicted her of shoplifting a $2,500 fur coat. She served eight months in a juvenile correction facility and thereafter was under the supervision of a parole officer for one year. After her parole, she completed high school, college, and law school, and she led a totally law-abiding life. When Sabrina applied for admission to practice law in State B, she was required to fill out a questionnaire. One question asked her to disclose "all convictions. including juvenile convictions." In answering that question she put "not applicable." on the theory that her juvenile offense in State A was irrelevant to her present moral character. The bar of State B did not learn about her State A conviction until six months after she had been admitted to practice in State B. Is Sabrina subject to discipline? A. Yes. because she withheld a material fact when she answered the questionnaire. B. Yes, because a person who has committed a crime involving dishonesty or false statement is disqualified from practicing law. C. No, because her prior juvenile conviction was not relevant to her moral character at the time of her application to the bar. D. No, because State B's questionnaire is an unconstitutional invasion of privacy.

6. B. Item I is incorrect; Item III is correct. Comment 14 to ABA Model Rule 1.8 states that lawyers are not prohibited from using an arbitration clause so long as the client is fully informed of the clause's scope and effect. Item II is not correct. Lawyers are entitled to insist that clients observe their fee agreements. ABA Model Rule 1.16(b)(5) permits a lawyer to withdraw if a client deliberately disregards the fee agreement. While it is true that a lawyer cannot leave a client in the lurch (shortly before trial, for example), and while a lawyer must take reasonable steps to protect the client's interests upon withdrawal, the question does not suggest that Aoki has acted improperly in refusing to do further work until she is paid. Item IV is correct. ABA Model Rule 1.8(h) states that a lawyer who seeks to settle a malpractice claim with a client must advise the client in writing of the desirability of seeking the advice of independent counsel in connection with the settlement, and must also provide a reasonable opportunity to obtain such counsel.

6. Attorney Aoki and client Cramer entered into a written agreement in which Aoki agreed to represent Cramer in a real estate venture in return for a specified hourly fee. The agreement provided that any malpractice or fee dispute would be arbitrated by a neutral arbitrator selected by mutual agreement. Eventually Aoki and Cramer did get into a dispute. Cramer refused to pay Aoki's quarterly bill, and Aoki refused to do any more work until Cramer paid. Cramer also threatened to sue Aoki for malpractice, claiming that he had lost money because of her negligent advice. Which of the following propositions are correct? I. Aoki is subject to discipline for trying to avoid a lawsuit for malpractice by including the arbitration provision in her contract with Cramer. II. Aoki is subject to discipline for refusing to do further work until Cramer paid her bill. III. It would be proper for Aoki to insist that Cramer abide by the arbitration provision in their contract. IV. If Aoki wants to settle her dispute with Cramer outside of the arbitration agreement, she must advise Cramer to obtain independent representation for that purpose. A. I, II, and IV only. B. III and IV only. C. II and III only. D. I and IV only.

6. B. An attempt to bribe a police officer strikes us as a clear example of conduct that shows dishonesty and unfitness to practice law, which makes the conduct grounds for discipline. [ABA Model Rule 8.4; Cal.Bus. & Prof.Code §6106.] Answer A is not correct; State C can discipline Arner even though his conduct took place elsewhere. [See ABA Model Rule 8.5; CRPC 1—100.] Answer C is not correct; the misconduct need not be connected with the practice of law to result in professional discipline. [See ABA Model Rule 8.4(b)—(d); Cal.Bus. & Prof.Code § 6106.] Answer D is not correct because it is too broad; not every criminal act is grounds for professional discipline. [1d,]

6. Attorney Arner is a member of the bar of State C. While on vacation in State D, Arner was stopped by a police officer for driving a rental car 95 mph in a 65 mph speed zone. Arner offered the police officer five crisp $100 bills, saying: "Do you think we can make this little problem go away?" Thereupon he was arrested for attempted bribery of a police officer, a felony. He was ultimately convicted of that offense in State D and was fined $10,000. Is Arner subject to discipline in State C? A. No, because his conduct took place beyond the jurisdiction of State C. B. Yes, because his conduct involves dishonesty and suggests that he is unfit to practice law. C. No, because his conduct was not connected with the practice of law. D. Yes, because commission of any criminal act is grounds professional discipline.

6. C. If Gresler personally hung around hospitals, passing out his professional cards to personal injury victims, he would violate ABA Model Rule 7.3. Likewise, he is subject to discipline for inducing other persons to do what he himself could not do. [See ABA Model Rule 8.4(a).] Further, many states prohibit lawyers from using "runners or cappers" to solicit legal business. [822, e.g., Cal.Bus. & Prof.Code §§ 6150—54.] Answers A and B are not correct for the reasons stated above. Answer D is not correct; no disciplinary rule prohibits Gresler from holding the seminar or dispensing accurate legal advice to those who attend.

6. Attorney Gresler offered a free half-day seminar for nurses, hospital attendants, and emergency medical personnel on personal injury law as it relates to accident victims. During the seminar, he told the group about the importance of preserving items of physical evidence, keeping accurate recordS of medical treatment, accurately recording statements made by the victim and others about the accident. and the like. At the close of the seminar, he passed out packets of his professional cards and invited. the members of the group to give them to accident victims. Was Gresler's conduct proper? A. Yes. because a lawyer has an ethical obligation to help non- lawyers recognize legal problems and handle those problems correctly. B. Yes. because his conduct is protected by the Free Speech clause of the First and Fourteenth Amendments. C. No. because he invited members. of the group to hand out his professional cards to accident victims. D. No. because he dispensed legal advice to people with whom he had no prior professional relationship.

6. C. Items A and D are incorrect; when Lee drew the $350 check on her client trust account, she was misappropriating Fujitomi's money. [ABA Model Rule 1.15.] Item B is incorrect; there is no authority for sanctioning Lee in this situation. Item C is correct; in these circumstances, the fine can be viewed as an expense of litigation, and lawyers are permitted to advance litigation expenses. [ABA Model Rule 1.8(e).]

6. Client Fujitomi entrusted lawyer Lee with $10,000, to be used six weeks later to close a business transaction. Lee immediately deposited it in her client trust account; at the time, it was the only money in that account. Later that same day, the local bar association called Lee and asked her to rush out to the Municipal Court to take over the defense of an indigent drunkard, Watkins, who was being tried for violating an obscure municipal statute. Because of chaos in the Public Defender's Office, Watkins was being tried without benefit of counsel. By the time Lee arrived, the judge had already found Watkins guilty and sentenced him to pay a fine of $350 or spend 30 days in jail. Under a peculiar local rule of court, the only way to keep Watkins from going to jail was to pay the fine immediately and to request a trial de nouo in the Superior Court. Therefore, Lee paid the fine with a check drawn on her client trust account, and Watkins promised to repay her within one week. Which one of the following statements is correct? A. Lee's handling of the Watkins matter was proper. B. Lee would have been subject to litigation sanction if she had allowed Watkins to go to jail. C. If Lee had paid Watkins' fine out of her personal bank account, that would have been proper. D. Lee would be subject to discipline for handling the matter in any manner other than she did.

6. B. Under ABA Model Rule 1.7(b), client consent solves the problem. Answer C is incorrect tbr the same reason that answer B is correct. Answer A is incorrect. because ABA Model Rule 1.7 makes no exception for the interest of a trust beneficiary. Answer D is. incorrect because the avm'lability of other counsel is irrelevant to this conflict of interest issue.

6. Client Parsons has asked lawyer Ekimoto to represent her, and nine other representatives of a plaintiff class, in an employment discrimination class action against Consolidated Telephone and Telegraph Corporation. The size of the plaintiff class and the size of the potential recovery are hard to estimate, but the case would conceivably produce a total recovery of nearly 15 million dollars. Lawyer Ekimoto and her two brothers are the beneficiaries of a trust fund established by their late parents. Among the trust assets are 1,000 shares of Consolidated common stock. Consolidated has 30 million shares of common stock outstanding. If Ekimoto reasonably believes that her interest in Consolidated will not affect her representation of the plaintiffs, may she serve as plaintiffs' counsel? A. Yes, since the interest of a trust beneficiary is not regarded as disqualifying. B. Yes, if she obtains the consent of the class representatives after full disclosure of her interest. C. No, since even a small adverse financial interest creates an appearance of impropriety. D. No, since there are other counsel available who could serve the plaintiff class without any potential conflict.

6. B. Federal Rule of Civil Procedure 11 applies at each stage of litigation whenever a lawyer advocates on the basis of written claims, defenses, or other legal contentions. Thus, when discovery reveals that a claim is frivolous, a lawyer is subject to Rule 11 sanctions for pursuing it, as with a motion for summary judgment. Answer D is wrong for the same reason. Answer A is wrong because ABA Model Rule 3.1 makes a lawyer subject to discipline for pressing a frivolous claim, no matter what the client's wishes. Answer C is wrong because it is too broad—victory in the underlying suit is an essential element of a malicious prosecution claim.

6. For a century or more, the commercial fishing industry in' Northport has been dominated by two feuding clans, the VonRutz family and the McCabe family. The McCabes hired lawyer Lang to sue the VonRutzes in federal court for predatory pricing in violation of the federal antitrust laws. The complaint alleges that the VonRutzes have been sellm'g their' fish below cost with the intent of driving the McCabes out of business, which would give the VonRutzes monopoly power in the Northport area. The information that was available to Lang when he drafted the complaint supported the "below cost" allegation. During pretrial discovery, however, it became obvious that the VonRutzes never sold their fish below their "average total cost." Under the applicable law, that means that they could not possibly have been engaged in predatory pricing, and no good faith argument can be made for changing that law. Nevertheless, the McCabes instructed Lang to move for summary judgment, explaining: "The VonRutzes caused misery for our fathers and our grandfathers and our great grandfathers. Winning isn't important—we just want to remind those rotten VonRutzes that it's expensive to mess with the McCabes." If Lang follows his clients' instruction to move for summary judgment: A. Lang will not be subject to discipline because he is obliged to follow his clients' instructions on matters that affect the clients' substantial legal rights. B. Lang will be subject to litigation sanctions because discovery has revealed that his clients' claim is frivolous. C. Lang will be subject to civil liability for malicious prosecution, no matter what the ultimate outcome of the predatory pricing case. D. Lang will be not be subject to discipline because he did not know until pretrial discovery that his clients' claim was frivolous.

6. B. ABA Model Rule 6.3 indicates that Leggett must not participate in the Board of Directors' decision. To participate and vote "yes" would be "incompatible" with his obligations to his firm's clients, a violation of Rule 6.3(a). To participate and vote "no" could "have a material adverse effect on the representation" of tenants who are the society's potential clients. [Cf. Hazard & Hodes § 52.4, Illustration 52—1.] Thus answers C and D are not correct. Answer A is not correct. It is true that ABA Model Rule 5.4(b) prohibits a lawyer from "forming a partnership" (or professional corporation) with a non-lawyer if any of the activities of the partnership (or professional corporation) consist of the practice of law. However, the boards of directors of legal aid societies typically include both lawyers and non-lawyers. Hazard and Hodes argue these regulations "sweep[] far too broadly." [Hazard & Hodes § 45.4.]

6. Lawyer Leggett is a partner in a private law firm that represents numerous landlords who rent apartments to low-income families. Leggett is also a member of the Board of Directors of the County Legal Aid Society. Some of the other directors are non-lawyers. The society offers free legal services to low-income clients in a variety of civil matters. The legal services are actually provided by paid lawyers on the society staff or by volunteer lawyers from the community. Up to now, the society has not had enough funding to offer services in landlord-tenant disputes. Recently it acquired a new source of funds, and now the Board of Directors needs to decide whether to add landlord-tenant disputes to the society's list of services. Leggett knows that an affirmative answer will adversely affect his firm's landlord clients. Which of the following is most nearly correct? A. Leggett is subject to discipline for being a director of a legal services organization in which some directors are non-lawyers. B. Leggett's participation in the decision will make him subject to discipline, no matter which way he votes. C. Leggett may participate in the decision, but if he votes "no," he will be subject to discipline. D. Leggett may participate in the decision, but if he votes "yes," he Will be subject to discipline.

6. A. Your duty here is to serve your client's best interests, and the way to do that is simply to call the mistake to Lauder's attention so it can be corrected. If your client signs the contract, knowing of the mistake, the contract is voidable; ultimately, your client may lose the benefit of the bargain, or end up in costly litigation, or both. [See RESTATEMENT (SECOND) OF CONTRACTS § 153 (1981).] Answers B and C are incorrect. It is true that you should not give up one of your client's valuable legal rights without first consulting the client and obtaining his consent. [See ABA Model Rule 1.2, Comments 1 and 2.] But you are not giving up any legal right here; your knowledge of the mistake is imputed to your client, and your client thus has no legal right to hold the other party to the mistaken version of the contract. Answer D is incorrect for a reason that you will study in a later Chapter. A lawyer must not communicate directly with a party on the opposite side of a matter if the lawyer knows that the party is represented by counsel. [ABA Model Rule 4.2.]

6. Over the past several months, you and lawyer Lauder have been representing your respective clients in a complicated contract negotiation. The proposed contract has been drafted, redrafted, and revised dozens of times during the negotiation. Finally, your respective clients have struck a bargain; their bargain includes a key provision that your client long resisted and ultimately accepted with great reluctance. The final version of the contract has been prepared by Lauder's secretary and has been signed by Lauder's client. Lauder has sent it to you for signature by your client. You have read it carefully, and you have discovered that the secretary left out the provision mentioned above. Which of the following may you do? A. Without consulting your client, call Lauder and direct her attention to the missing provision. B. Call your client, explain the situation, and do whatever your client directs. C. Call your client, explain the situation, and advise him to sign the contract. D. Call Lauder's client and ask whether he ultimately decided not to insist on the provision.

6. C. ABA Model Rule 1.6(a) requires Arnott to keep Curtis's information in confidence, unless Curtis changes her mind and consents to have it revealed. [See Michigan State Bar Op. 01—1141 (1986).] Answer A is not correct; Arnott has no discretion here. Answer B is not correct. The holder of the attorney-client privilege (and the beneficiary of the ethical duty) is Curtis; whether or not Coleman's evasion of the law is a continuing crime, Curtis is still entitled to the protection of confidentiality. Answer D is not correct. Arnott would be breaching the duty of confidentiality even if he asked the prosecutor not to reveal the source of the information. The prosecutor might not comply with the request. Even if the prosecutor does, Coleman or one of the other friends of Curtis's ex-husband may be able to figure out where the information came from, thus putting Curtis and her children in danger.

6_ Eight years ago, attorney Arnott represented client Coleman in connection with a murder investigation. Coleman repeatedly assured Arnott that he was innocent. The investigation proved futile, and Coleman was never formally charged with any crime. At present Arnott is representing client Curtis in a child custody dispute between Curtis and her ex-husband. In that connection, Curtis tells Arnott in confidence about a murder committed eight years earlier by one Coleman, a friend of her ex-husband. The details revealed by Curtis make it clear that Arnott's former client, Coleman, did commit the murder. Curtis insists that Arnott not tell anyone about the murder for fear that Coleman or some of her ex-husband's other friends may retaliate against her or her children. This jurisdiction has no statute of limitations on murder. Which of the following is most nearly correct? A. Arnott may reveal the information to the prosecutor without Curtis's consent, because this jurisdiction has no statute of limitations on murder. B. Arnott must reveal the information to the prosecutor because Coleman's evasion of the law is a continuing crime. C. Arnott must keep the information in confidence unless Curtis changes her mind and consents to have it revealed. D. Arnott may reveal the information to the prosecutor without the consent of either Curtis or Coleman, provided that he asks the prosecutor not to disclose the source of the information.

7. C. See Ohio Supreme Court Board of Commissioners on Grievances and Discipline Op. 99—9 (1999), in which the Ohio ethics panel approved an e-mail question answering scheme similar to this one. Answer A is wrong because there is no rule against including personal information in a lawyer's promotional material, even if the personal information is irrelevant to the selection of a lawyer. Answer B is wrong because there is no rule against dispensing legal advice to a person with whom you have no prior relationship. Besides, we think that answering the e-mail inquiry would establish a lawyer-client relationship with the questioner, for purposes of malpractice law and the like. Answer D is wrong because no rule prohibits a lawyer from speaking or writing to the public about controversial legal issues or about legal issues that require specialized knowledge.

7. After graduating from law school, three young women formed their own new firm dedicated to the law of women's rights in the workplace. They established an attractive site on the Internet. Their website includes very specific biographical information about each of them, including information about their families. their hobbies, and all the academic and athletic honors they received in college and law school. The site also includes detailed, thoroughly researched position papers they have written on current legal issues in their field of law practice. The position papers are written so they can be understood by a lay audience. The website includes an e-mail link that allows site visitors to ask legal questions of the three lawyers. The question form requires the questioner to supply enough information about him or herself to permit the lawyers to do a conflict of interest check before responding to the question. The questioner supplies a credit card number, and the lawyers charge a small fee for e-mailing an answer to the question. The lawyers do not answer questions from site visitors who live outside the state in which the lawyers have their office. If a question is too difficult to answer competently by e-mail, the lawyers invite the questioner to come to their office for a free initial consultation. Is the conduct of the three lawyers proper? A. No, because their website includes personal information about the three lawyers that is not relevant to the potential client's selection of a lawyer. B. No, because the e-mail feature permits the three lawyers to dispense legal advice to people they have never met and with whom they have never established a lawyer-client relationship. C. Yes, but only if the fee for emailing an answer to a question is not unreasonably high. D. Yes, but only if the position papers are on non-controversial legal issues that do not require a specialized knowledge of women's employment law.

7. D. Under ABA Model Rule 1.8(1'), a lawyer must not acquire an ownership interest in the subject of litigation that the lawyer is conducting for the client. (subject to two exceptions that do not apply here). The subject of this declaratory judgment action is Belloni's United States Patent. and Lothrop is acquiring a 10% interest in that patent. Answer A is incorrect for the same reason that answer D is correct. Answer B is incorrect because a lawyer may require the client to repay advanced litigation expenses, no matter what the outcome of the litigation. [See ABA Model Rule 1.8(e)(1); Hazard & Hodes §12.12.] An'swer C is incorrect because a lawyer may advance litigation expenses in a civil case. [Id.]

7. Biochemist Belloni invented a gene splicing process for making snake antitoxins. The invention was a major breakthrough because Belloni's antitoxins were far cheaper and more reliable than the natural variety produced from the venom of live snakes. She obtained a US Patent on her process. Shortly thereafter, she was sued in a declaratory judgment action brought by United Laboratories, Inc. United sought a declaration that her US. Patent was invalid. Belloni asked lawyer Lothrup to represent her in the case. Lothrup agreed to do so on the following terms: (1) Belloni would pay Lothrup for the necessary legal work at Lothrup's regular hourly rate; (2) Lothrup would advance the litigation expenses, subject to repayment by Belloni no matter what the outcome of the case; and (3) at the outset, Belloni would assign to Lothrup a 10% ownership interest in the US. Patent. A. The arrangement is proper, assuming that the total Lothrup earns from it is reasonable. B. Lothrup is subject to discipline because the arrangement requires Belloni to pay back the advanced litigation expenses even if she loses the declaratory judgment case C. Lothrup is subject to discipline because the arrangement prov1des for an advance of litigation expenses by the lawyer in a civil case. D. Lothrup is subject to discipline because the arrangement gives her a personal financial interest in the US. Patent which is the subject of the declaratory judgment case

7. A. ABA Model Rule 8.3 requires a lawyer to report serious misconduct by another lawyer, but that duty does not apply where the first lawyer learns about the misconduct through a privileged communication. Lindell's communication with Cathcart was subject to the attorney-client privilege, and none of the exceptions to the privilege would apply in this situation. Thus, Lindell has no duty to reveal the information, and if he did reveal it, he himself would be subject to discipline. [See ABA Model Rule 1.6.] Answers B, C, and D are not correct because each of them involves some form of prohibited disclosure of the confidential information.

7. Client Cathcart hired lawyer Lindell to prepare an estate plan. In connection with that work, Cathcart told Lindell in strict confidence about a criminal fraud perpetrated by Cathcart's former lawyer, Foreman. Lindell urged Cathcart to report Foreman's conduct to the state bar. For unstated reasons, Cathcart refused to do so and refused to allow Lindell to do so. What is the proper course of conduct for Lindell in this situation: A. To keep the information instructed. B. To speak with Foreman in confidence, Cathcart said, and to urge Foreman to rectify his fraud. C. To report the information to the state bar, despite Cathcart's instruction not to do so. D. To write an anonymous letter to the state bar, relating the facts disclosed by Cathcart

7. A. Clark and Craddock were joint clients of Ling. As joint clients, both of them were holders of the attorney-client privilege. But in litigation between two former joint clients, neither of them can claim the attorney-client privilege. [See MCCORMICK ON EVIDENCE §91.1 at 158 (Hornbook ed. 2006).] Therefore, the court was correct in ordering Ling to disclose what Clark said. Having been properly ordered by the court to disclose the information, Ling must do so. [See ABA Model Rule 1.6, comment 3 ("The rule of client-lawyer confidentiality applies in situations other than those where evidence is sought from the lawyer through compulsion of law.").] Further, it was proper for Ling to withdraw as counsel for Clark and Craddock because of the conflict between their interests. [ABA Model Rule 1.16(a).] Answer B is not correct; as noted above, Ling must disclose what Clark said. Answer C is not correct. Ling's withdrawal was proper, and she must disclose what Clark said. Answer D is not correct; again, having been ordered to answer by the court, Ling must do so.

7. Lawyer Ling represented clients Clark and Craddock who were the sole partners in a business joint venture. In that connection, Clark and Craddock met frequently with Ling to discuss confidential matters relating to the business. One day Clark came alone to Ling's office. Before Ling could stop him, Clark disclosed that he had usurped a business opportunity that properly belonged to the joint venture. Ling informed Clark that she could not advise him on that topic. Further, Ling promptly withdrew as counsel to Clark and Craddock. Ultimately Craddock sued Clark for the usurpation. Craddock's lawyer subpoenaed Ling to testify at a deposition about the statements Clark made to Ling. At the deposition, Clark's lawyer asserted the attorney-client privilege on Clark's behalf. Ultimately the court ordered Ling to disclose what Clark said. Which of the following is most nearly correct? A. It was proper for Ling to withdraw as counsel to Clark and Craddock. Further, Ling must disclose what Clark said. B. It was proper for Ling to Withdraw as counsel to Clark and Craddock. However, Ling will be subject to discipline if she discloses what Clark said. C. Ling is subject to discipline for withdrawing as counsel to Clark and Craddock. Further, Ling will be subject to discipline if she discloses What Clark said. D. Even if Ling believes that the court order is correct, she must refuse to disclose what Clark said.

7. A. Item I is not correct. It is not uncommon for a lawyer to have a non-lawyer employee handle the day-to-day details of the client trust account, and no legal ethics rule forbids it. Items II and III are correct. ABA Model Rule 5.3 requires lawyers to train and adequately supervise their non-lawyer assistants. In re Scanlan, 144 Ariz. 334, 697 P.2d 1084 (1985), imposed discipline on a lawyer for conduct similar to that described in this question. Likewise, if Pearce was negligent in her supervision of Nelson, then Pearce is liable for malpractice under ordinary principles of respondeat superior. Item IV is not correct. Pearce's subjective, good faith belief is beside the point. The standard is an objective one: if a reasonably prudent lawyer would not have allowed Nelson to handle the client trust account, considering the facts stated in the question, then Pearce has breached her duty of care.

7. Solo practitioner Pearce hired non-lawyer Nelson to serve as her secretary and all-purpose assistant. Pearce put Nelson in charge of her client trust account and her office account and instructed her about how the accounts were to be handled. Several months later, Pearce learned that Nelson had a criminal record, including two prior convictions for embezzlement from a former employer. Since Nelson appeared to be handling the accounts properly, Pearce decided to leave well enough alone. After Several more months, Pearce noticed that $1,500 was missing from the office account. Nelson explained that she had borrowed the money to pay her mother's funeral expenses and that she would repay it out of her next paycheck. Nelson did repay the money, and Pearce decided to let Nelson continue to manage the accounts. Then, a year later, Nelson disappeared along with $30,000 from Pearce's client trust account. The clients whose money was taken sued Pearce for negligence and breach of fiduciary duties. Which of the following propositions are correct? I. Pearce is subject to discipline for allowing a non-lawyer to handle her client trust account. II. If Pearce did not adequately supervise Nelson's handling of the client trust account, then Pearce is subject to discipline. III. Pearce is subject to civil liability for malpractice to the injured clients if she was negligent in allowing Nelson to handle the client trust account. IV. If Pearce had a subjective, good faith belief that Nelson was trustworthy, then Pearce is not subject to civil liability for malpractice to the injured clients. A. II and III only. B. I, II, and III only. C. IV only. D. II and IV only.

7. A. On a somewhat similar set of facts, a California court disqualified the entire county prosecutor's office. [People v. Lepe, 164 Cal.App.3d 685, 211 Cal.Rptr. 432 (1985). But see People U. Hernandez, 235 Cal.App.3d 674, 286 Cal.Rptr. 652 (1991) (limiting disqualification to lawyers who personally participated with former witness, now a defendant).] If Aldrich did a competent job as defense counsel in the two assault and battery cases, she undoubtedly talked with Costa in detail about his relationship with his brother-in-law, Vincent. The information she received as Costa's defense lawyer could become relevant in the murder prosecution—for example, to prove motive, intent, or premeditation. Thus, Aldrich herself would be barred from personally prosecuting Costa, and answer B is accordingly incorrect. Answer C is incorrect for the same reason; her "direct, immediate and personal supervision" would involve the same kind of conflict as if she personally prosecuted Costa. Answer D is incorrect because If she screens herself off from the case, she cannot exercise the "direct, immediate, and personal supervision" that the state statute mandates. We believe that ABA Model Rule 1.11(d) does not apply to this problem, because the state statute allows the State Attorney General to take over cases in which the local prosecutor's office has a conflict of interest.

7. When attorney Aldrich was in private practice, she defended client Costa in two criminal assault and battery cases. The cases were three years apart, and both times the victim was Vincent, Costa's brother-in-law. Costa was convicted in both cases. Thereafter, Aldrich was elected County Prosecutor. As County Prosecutor, Aldrich hires and fires deputy prosecutors and generally supervises their work. As time permits, she also personally prepares and tries some cases. Her former client Costa is in trouble again, this time for the apparent first-degree murder of Vincent. A state statute requires all first-degree murder prosecutions to be conducted under the "direct, immediate, and personal supervision" of the County Prosecutor. The statute further provides that the State Attorney General's Office shall take over any criminal prosecution in which the local County Prosecutor cannot act due to conflict of interest. Which of the following is most nearly correct? A. Aldrich may ask the State Attorney General's Office to take over the prosecution because she and her deputies have a conflict of interest. B. Aldrich may personally prepare and prosecute the case because it arises out of a transaction separate and distinct from those in which she represented Costa. C. Aldrich may assign one of her deputies to prepare and prosecute the case, so long as she undertakes "direct, immediate, and personal supervision" of the work. D. Aldrich may assign one of her deputies to prepare and prosecute the case and then carefully screen herself off from any personal participation in the case.

8. A. The conflict of interest here is between Andrate (who wants to be paid for his work) and his indigent inmate clients (who want the medical treatment that the consent decree will give them). [See ABA Model Rule 1.7(a).] All who have written about this issue agree on one basic point: the decision whether to accept or reject the settlement offer is for the client to make, not for the lawyer to make. [See ABA Model Rule 1.2; see generally Judith L. Maute, Allocation of Decisionmaking Authority Under the Model Rules of Professional Conduct, 17 U.C.Davis L.Rev. 1049 (1984).] Answers B, C, and D are incorrect because in each instance the decision is made by the lawyer rather than the client. Evans v. Jeff0., 475 US. 717, 106 S.Ct. 1531, 89 L.Ed.2d 747 (1986), concerns the settlement of a civil rights class action. A federal statute permits (but does not require) an award of attorney fees to the prevailing party in such an action. The defendant offered to settle for a consent decree that would give the plaintiffs more injunctive relief than they probably would get by going to trial, provided that the plaintiffs would not seek an award of attorney fees. The issue before the Court was whether the trial judge was correct in approving the settlement containing the fee waiver. In a 6—3 decision, the Court said the trial judge was correct. Since Congress did not require fee awards, but only permitted them, the fee waiver was not antithetic to the purpose of the fee award statute. Further, to forbid fee waivers would impede settlements, because defendants naturally wish to clean up both the substantive issues and the fee issues at the same time. The dissenting opinion points out that to permit fee waivers will make attorneys far less willing to represent indigent plaintiffs in difficult civil rights cases. Query whether it is proper for a defendant to insist on a fee waiver as a condition of settlement. [See Ass'n Bar of City of New York Opinion 1987—4 (1987); Hazard & Hodes §§ 8.19—8.20]

8. A statute of State X requires prison "sanitary living conditions, suitable education and recreation facilities, and competent medical treatment." The statute authorizes inmates who are deprived of these benefits to sue the State Commissioner of Prisons for equitable relief. The statute also permits (but does not require) the courts to order State X to pay the attorney fees of successful inmate plaintiffs. At the request of the local bar association, private attorney Andrate agreed to represent a group of indigent inmates who were allegedly being deprived of proper medical attention at a State X prison. After extensive discovery proceedings, the State Commissioner of Prisons offered to settle the case by entering into a consent decree that would give the inmates all the equitable relief they could ever hope to receive, provided that An'drate would not request an award of attorney fees. Which of the following would be the proper thing for Andrate to do with respect to the settlement offer? A. Explain it to his clients and let them decide whether to accept it or reject it. B. Reject it on behalf of his clients because it does not provide for an award of attorney fees. C. Accept it on behalf of his clients, even though it does not provide for an award of attorney fees. D. Reject it on behalf of his clients because to do otherwise would discourage private attorneys from representing indigent inmates in future cases.

8. C. Item IV is correct. Liggett can be disciplined for ratifying Prentice's misconduct and for failing to take steps to mitigate its consequences. [ABA Model Rule 5.3(c).] Item I is not correct. Searching through files for documents that have to be produced is the kind of task that is often delegated to a non-lawyer assistant; the delegation is proper so long as the lawyer adequately supervises the non-lawyer's work and takes ultimate responsibility for it. [See Comment to ABA Model Rule 5.3.] Item III is not correct. A lawyer is subject to discipline for harassing the adversary in discovery proceedings. [See ABA Model Rule 3.4(a), (c), and (d).] Fed.R.Civ.P. 34(b) states that "a party who produces documents for inspection shall produce them as they are kept in the usual course of business or shall organize and label them to correspond with the Cc'ltt'gOI'it'S in the request." lf Liggett himself had jumbled the documents, he could be StlllCllOIled under Fed.R.(.»'iv.l'. 37 as well as disciplined by the bar. Here it was Prentice who jumbled the documents, but Liggett apparently ratified her conduct. or at least failed to take steps to mitigate its consequences. [ABA Model Rule 5.3(c).] Item II is not correct for the same reasons; it was not enough for Liggett to tell Prentice not to do such things in the future. He should have taken steps to mitigate the consequences of her misconduct. [Id.] For example, he might instruct her to put the documents back in the proper order; if that is impossible, perhaps he would have to flag the harmful documents so that the adversary could find them.

8. Supervising lawyer Liggett assigned paralegal Prentice to search through the massive business files of Liggett's client to find documents responsive to a federal court order for production of documents. After several months" work, Prentice ended up with 170 large cartons full of documents that were responsive to the court order. Most of the documents were harmless, but a few were quite damaging to the legal position taken by Liggett's client. Instead of arranging the documents in the same logical order in which she found them in the client's files, Prentice intentionally jumbled the order of the documents. Her purpose was to make it exceedingly difficult, if not impossible, for the adversary to find the damaging documents and to understand their significance. Before the documents were produced for the adversary. Prentice told Liggett what she had done. Liggett responded: "Good—that ought to slow the bastards down. In the future, however, don't do anything like that without checking with me first; we might get in trouble otherwise." Which of the following statements are correct? I. Since document production requires the skill and judgment of a lawyer, Liggett is subject to discipline for delegating the task to Prentice, even if he had adequately supervised her work II. Liggett's conduct was proper Since he admonished instructed her not to engage in similar conduct in the future. III. Assuming that all responsive documents were produced, Liggett's conduct was proper, since the adversary has no right to insist that the documents be arranged in any particular order. IV. Even if all responsive documents were produced, Liggett is subject to discipline because he failed to take steps to mllgate the consequences of Prentice's misconduct. A. II only. B. I and IV only. C. IV only. D. II and III only.

8. A. ABA Model Rule 7.1 prohibits misleading communications about legal services. The term "affiliate" is broad and vague, and it has been applied to many kinds of relationships between law firms. ABA Formal Opinion 94—388 (1994) says that when attorneys use "affiliate" on a letterhead or similar advertising, they must take the additional step of explaining precisely what they mean by it. The additional explanation need not be given to everyone; it is sufficient to give it to those prospective clients who may care about it. Answer B is wrong, for the reason stated above. Answer C is wrong because there is no such rule. Answer D is wrong because the referral of work by one of these firms to the other does not violate any rule about solicitation of clients.

8. The firm of Wilkens & Crosse has existed for many years in Chicago. Now it wishes to open an office in Los Angeles. The Los Angeles office will be established as a separate partnership. Some of the proposed L08 Angeles partners are admitted to practice only in California, and they will not become partners in the Chicago firm. Some of the Ch1cago partners are admitted to practice in both Illinois and California; they will retain their partnership in the Chicago firm, and they will also become partners in the Los Angeles firm. The letterheads of both firms will accurately identify which lawyers are admitted to practice in which jurisdictions. The two firms will regularly refer work back and forth, and each firm will be available to the other firm and its clients for consultation and advice. Further, on some occasions, partners and associates will be transferred from one firm to the other. Each firm will advertise itself as an "affiliate" of the other firm. Is the arrangement proper? A. Yes, provided that the nature of the "affiliate" relationship is explained. B. Yes, because "affiliate" is a broad term that can cover many kinds of relationships. C. No, because a partner of a firm in one state is not permitted to be a partner of a firm in a different state. D. N 0, because the arrangement contemplates the referral of work from one firm to the other in violation of the solicitation rule.

1. C. ABA Model Rules 5.1 and 5.2 apply here. A variety of federal and state statutes prohibit the destruction of subpoenaed documents. [See generally WOLFRAM, MODERN LEGAL ETHICS § 12.3.5 (1986).] When Crawford shredded the files and later erased the computer backup, he doubtless violated one or more of these statutes; further, he is subject to professional discipline. [See ABA Model Rule 3.4(a).] Even if Crawford thought that Barker had instructed him to destroy the material, Crawford is still subject to discipline—the ethical duty here is too clear to be subject to reasonable argument. [Compare ABA Model Rule 5.2(a) with 5.2(b).] Barker is also subject to discipline. When Barker learned that Crawford had shredded the files, Barker should have acted promptly to preserve the computer backup; instead, Barker made an ambiguous comment that an overzealous young lawyer might interpret as an instruction to erase the computer backup. [See ABA Model Rule 5.1(c)(2).] Altmont is subject to discipline for lying to the congressional committee about the destruction of the files and computer backup [ABA Model Rule 3.3(a)(1)], and perhaps also for failing to make reasonable efforts to assure that Barker and Crawford would act ethically [ABA Model Rule 5.1(b)] or for ratifying their unethical conduct [ABA Model Rule 5.1(c)(1)].

A congressional investigating committee subpoenaed certain files from a governmental agency in connection with the committee's investigation of the agency's allegedly illegal expenditure of government funds. Lawyer Altmont (the agency's Chief Counsel) instructed lawyer Barker (the Deputy Chief Counsel) to gather up the files and prepare them for production. Barker, in turn, assigned the project to lawyer Crawford (a newly-hired junior lawyer). In giving Crawford the assignment, Barker said: "I Wouldn't be surprised if all of these files have been shredded long ago. pursuant to our regular Document Storage and Retention Procedures Manual ('DSRI'M')." Crawford discovered that the files still existed. even though the llSltl'M called for their destruction six months earlier. Crawford dutifully shredded the files himself and then reported the fact to Barker. Barker responded by stating: "Good. I wonder if the computer backup for those files still twists?" Crawford interpreted this as an instruction to erase the computer backup material, which he promptly did. Barker then reported the full story to Altmont who informed the congressional investigating committee that both the files and the computer backup had been destroyed in accordance with the agency's regular procedures under the DSRPM. A. Only Barker and Crawford are subject to discipline. B. Only Crawford is subject to discipline. C. Altmont, Barker, and Crawford are all subject, to discipline. D. Neither Altmont, nor Barker. nor Crawford are subject to discipline.

7. C. Canfield authorized Ayers to accept any reasonable settlement offer and said, "try to get the horse if you can." Note that Canfield sued for specific performance, not for return of the bonds— Canfield obviously has a higher opinion of Thunderbolt than Ayers does. At the outset, Ayers could properly have counseled Canfield to seek the bonds rather than the horse. [See ABA Model Rule 1.2(a) and the Comment thereto] But the time for counseling has passed; Ayers" duty at this point is to carry out his client's instructions. Thus, answers A and B are not correct. When Ayers gets the horse from Dennis, he must keep it in a safe place until Canfield returns. [ABA Model Rule 1.15(a).] The bonded stable is a better choice than the pasture on Ayers' farm, mentioned in answer D; who knows what evil might befall Thunderbolt out in Ayers" pasture? The bonded stable will cost nearly $1,000, but under the circumstances it is proper for Ayers to incur this expense and to seek reimbursement from Canfield. [See RESTATEMENT (SECOND) OF TRUSTS § 176 (1959); A. SCOTT, TRUSTS § 176 (3d ed. 1967); R. BROWN, LAW OF PERSONAL PROPERTY § 11.10 (3d ed. 1975).]

Attorney Ayers represents client Canfield as plaintiff in a suit to compel specific performance of a contract. Canfield contracted to purchase Thunderbolt, a thoroughbred race horse, from defendant Dennis in exchange for $1,500,000 worth of corporate bonds owned by Canfield. Canfield transferred the bonds to Dennis, but Dennis refused to deliver Thunderbolt. Two months before the scheduled trial date, Canfield gave Ayers the following instructions: "I am leaving tomorrow on a six week sailing trip through the SOuth Pacific, and you will not be able to reach me by any means. If Dennis makes any reasonable settlement offer before I return, please accept it, but try to get the horse if you can." A week later, Dennis' lawyer called Ayers and said: "Dennis wants to capitulate. He will either return the bonds, or he will turn over Thunderbolt. He insists on an immediate response, so call me back this afternoon." Ayers believes in good faith that Thunderbolt is a tired nag, worth far less than $1,500,000. Further, Ayers discovers that it will cost nearly $1,000 to keep Thunderbolt in a safe, bonded stable until Canfield's return. What is the proper course of action? A. Get the bonds and put them in a safe deposit box until Canfield returns B. Tell Dennis' lawyer that he cannot respond until Canfield returns C. Get Thunderbolt and house him in the safe, bonded stable at Canfield's expense until Canfield returns D. Get Thunderbolt and turn him out to pasture on Ayers' farm until Canfield returns

E

PK 1.

A

PK 10.

D

PK 11.

B

PK 12.

B

PK 13.

D

PK 14.

C

PK 2.

B

PK 3.

A

PK 4.

D

PK 5.

A

PK 6.

C

PK 7.

D

PK 8.

D

PK 9.


Conjuntos de estudio relacionados

Rosetta Stone French Unit 5 Lessons 1-3

View Set

Section 3: Money and the Monetary System

View Set

1. Occipital Lobe and Parietal Lobe

View Set

AP Gov - Test Questions: Chapter 6

View Set

Securities Industry Essentials (SIE) Practice Test

View Set